Important Announcement
PubHTML5 Scheduled Server Maintenance on (GMT) Sunday, June 26th, 2:00 am - 8:00 am.
PubHTML5 site will be inoperative during the times indicated!

Home Explore Endo 19 4e

Endo 19 4e

Published by vgu08417, 2020-05-23 10:46:03

Description: Endo 19 4e

Search

Read the Text Version

Cas cliniques Question 15 Question 3 Le bilan fait au diagnostic n'a rien montré de particu- Vous avez finalement diagnostiqué un diabète lier : examen ophtalmologique normal, ECG normal, chez la patiente. Vous dosez son HbA1c, qui est épreuve d'effort normale, doppler des carotides nor- à 8,1 %. mal, soins dentaires en cours, disparition de l'adéno- Votre cible thérapeutique globale chez cette patiente pathie cervicale et examen ORL normal. Il a pris 6 kg comprend l'(les) élément(s) suivant(s) : de poids et se porte bien. A une HbA1c inférieure à 7 % Quel(s) examen(s) souhaitez-vous obtenir 6  mois B une normalisation pondérale après le diagnostic ? C un niveau de pression artérielle systolique en A examen ophtalmologique automesure inférieure à 139 mm Hg B mesure du rapport albumine/créatinine urinaire D une albumine urinaire inférieure à 300 mg/24 heures C dosage de l'HbA1c E un arrêt complet du tabac D dosage de la TSH Question 4 E recherche des anticorps anti-thyréoperoxydase Vous avez mis en route une modification thérapeu- tique du mode de vie et, après 3 mois, la patiente a une HbA1c à 7,4  % et une créatinine plasmatique Cas clinique 16 tout à fait normale à 53 μmol/l. Vous proposez la (les) mesure(s) suivante(s) : Madame I., 42  ans, consulte son médecin traitant A intensification des mesures diététiques car elle a entendu une émission de radio sur le B mise en route d'un sulfamide hypoglycémiant diabète et s'inquiète car elle pense qu'elle pour- C mise en route d'un biguanide rait avoir cette maladie. Elle n'a pas de problème D mise en route d'un inhibiteur de la DPP-4 évident, ne prend pas de médicament, a une E augmentation de l'activité physique contraception par DIU et n'a pas d'antécédents 458 particuliers (deux grossesses il y a 14 et 10 ans avec Question 5 accouchement normal à terme par voie basse d'un garçon à chaque fois de 3 700  g et 3 840  g). Elle Vous décidez de proposer à la patiente la mise en fume très modérément (3  cigarettes par jour) et parfois un peu plus en atmosphère festive. Elle pèse route d'un traitement par metformine. 70 kg pour 1,71 m (IMC = 23,9 kg/m2) et sa PA est de 123/76 mm Hg. Il n'y a pas de diabète dans sa Quelle(s) est (sont) la (les) proposition(s) exacte(s) ? famille (« mais on ne sait jamais… »). A risque d'hypoglycémie fréquent B effet indésirable digestif fréquent Question 1 C mise en route à une dose minimum et titration tri- mestrielle selon HbA1c D précautions d'emploi en cas de déshydratation Vous expliquez à la patiente qu'elle n'a pas besoin E action via un effet sur le récepteur au glucagon de dépistage du diabète. Cet avis se fonde sur l'(les) Question 6 Un an plus tard, vous n'avez pas franchement pu élément(s) suivant(s) : améliorer les choses : l'HbA1c reste à 7,6 % le jour de A indice de masse corporelle normal votre consultation contre 7,5 % 3 mois plus tôt. Vous B absence d'antécédent familial souhaitez modifier la thérapeutique pour améliorer C âge inférieur à 60 ans l'équilibre glycémique. D absence d'hypertension artérielle En cas d'échec de la monothérapie par metfor- E absence d'anomalie lors des grossesses mine, vous proposez pour cette patiente la (les) Question 2 modification(s) suivante(s) : La patiente a suivi vos conseils et vous redemande A remplacement de la metformine par un inhibiteur 4 ans plus tard (46 ans) de dépister son diabète, car elle a bien compris ce que vous lui aviez expliqué la de l'α-glucosidase première fois. Entre-temps, elle a présenté un épisode B adjonction d'un sulfamide hypoglycémiant dépressif, a pris du poids (80  kg maintenant, IMC C remplacement de la metformine par un inhibiteur = 27,3 kg/m2) mais va plutôt bien. Vous faites le dépistage du diabète de cette patiente de la DPP-4 selon la (les) procédure(s) suivante(s) : D chirurgie bariatrique, après prise en charge pen- dant 6 mois E proposition d'un traitement par insuline A une glycémie à jeun répétée dans les 15 jours en Question 7 cas de résultat entre 1,10 g/l et 1,25 g/l Vous avez donc opté pour le maintien de la met- B une glycémie à jeun et une mesure de glycémie formine à pleine dose et l'ajout d'un sulfamide 2 heures après HGPO 75 g C une glycémie à jeun à ne pas répéter si elle dépasse hypoglycémiant. 2 g/l Quelle(s) est (sont) la (les) proposition(s) exacte(s) ? D une glycémie à jeun répétée dans les 15 jours en A risque d'hypoglycémie fréquent B effet indésirable digestif fréquent cas de résultat entre 1,26 et 1,99 g/l C titration trimestrielle selon l'HbA1c E un dosage d'hémoglobine glyquée

Cas cliniques 25 D conseils pour débuter une autosurveillance Quelle(s) est (sont) la (les) proposition(s) exacte(s) ? 459 glycémique A réduction des doses de sulfamides B arrêt de la metformine Entraînement E action via un effet sur le canal chlore de la cellule C substitution du sulfamide par un inhibiteur de la bêta DPP-4 Question 8 D substitution de la metformine par un inhibiteur de La patiente vous interroge car elle se sent faible les après-midi, notamment le mardi. Vous évoquez la DPP-4 une hypoglycémie et recherchez une circonstance E substitution du sulfamide par un inhibiteur de favorisante. Vous avancez dans votre diagnostic avec les éléments l'α-glucosidase suivants : A glycémie capillaire lors des sensations de faiblesse Question 13 B recherche des apports en glucides habituels Le diabète de Madame  I. est bien équilibré dans les C interrogatoire sur le menu du mardi notamment années suivantes et elle revient vous voir régulière- D recherche d'un stress professionnel particulier le ment, suivant bien vos conseils et ceux de l'équipe de diabétologie avec laquelle vous collaborez régulière- mardi ment. Madame I. a 56 ans, une durée de diabète de E recherche d'une activité physique moindre les mardis 10 ans, son traitement actuel contient sulfamides et metformine à pleine dose. Son HbA1c est en dégra- Question 9 dation régulière malgré un suivi correct des modifi- Vous sentez que le cas de Madame  I. est délicat et cations du style de vie, même si l'IMC est à 32 kg/m2 imaginez plusieurs raisons à son équilibre glycémique maintenant. Les deux derniers dosages sont à 8,2 % qui ne s'améliore pas vraiment. et 8,5  %. Sa fonction rénale est tout à fait nor- Vous envisagez donc l'(les) option(s) suivante(s) : male à 92  ml/min. Vous proposez une modification A avis spécialisé du diabétologue thérapeutique. B dosage de la TSH Quelle(s) est (sont) la (les) proposition(s) exacte(s) ? C dosage de FSH A adjonction à la thérapeutique actuelle d'un ago- D reprise des conseils diététiques et de l'activité physique E proposition d'un programme d'éducation théra- niste du GLP-1 B adjonction à la thérapeutique actuelle d'un peutique glinide Question 10 C adjonction à la thérapeutique actuelle d'un inhibi- Vous reprenez à tête reposée le dossier de Madame I. Elle a 48 ans, un diabète depuis l'âge de 46 ans qui teur de la DPP-4 est difficile à équilibrer sous metformine seule. Les D adjonction à la thérapeutique actuelle d'un traite- choses se sont améliorées depuis la mise en route du sulfamide hypoglycémiant, mais il persiste tou- ment par insuline jours des hypoglycémies probables les après-midi. La E adjonction à la thérapeutique actuelle d'un inhibi- patiente a vu son poids passer de 80 à 85 kg malgré ses efforts sur le plan diététique. teur de l'α-glucosidase Concernant cette classe médicamenteuse, quelle(s) est (sont) la (les) proposition(s) exacte(s) ? Question 14 A risque d'hypoglycémie fréquent Vous avez décidé d'associer metformine pleine dose, B effet indésirable digestif fréquent sulfamide pleine dose et agoniste du GLP-1. C prise de poids fréquente Au sujet de cette classe thérapeutique, quelle(s) est D coût élevé des médicaments (sont) la (les) proposition(s) exacte(s) ? E effet insulino-sécréteur A contre-indication en cas d'insuffisance rénale Question 11 sévère Concernant cette classe médicamenteuse, quelle(s) B contre-indication en cas de pancréatite aiguë est (sont) la (les) proposition(s) exacte(s) ? C contre-indication en cas d'injection de produit de A association à l'insuline possible B association aux agonistes du GLP-1 contre-indiquée contraste iodé C association aux inhibiteurs de la DPP-4 possible D contre-indication en cas d'insuffisance respiratoire D association aux glinides contre-indiquée E contre-indication en cas de traitement par AVK E association aux inhibiteurs de l'α-glucosidase Question 15 possible Toujours au sujet des agonistes du GLP-1, quelle(s) est (sont) la (les) proposition(s) exacte(s) ? Question 12 A il s'agit d'un insulino-sensibilisateur La dernière HbA1c est à 6,1  % mais la prise de B ces médicaments s'associent généralement à une poids associée aux hypoglycémies de Madame  I. vous questionne. Vous proposez une modification perte de poids thérapeutique. C ces médicaments empêchent la production de sucres simples dans l'intestin D ces médicaments provoquent des hypoglycémies fréquentes E il faut indiquer à la patiente le risque de pancréa- tite aiguë

Cas cliniques Cas clinique 17 Quel(s) autre(s) facteur(s) additionnel(s) aurai(en)t pu Une femme âgée de 28 ans est admise aux urgences concourir à cette décompensation ? pour une douleur abdominale aiguë intense apparue A la survenue d'un diabète depuis 6  heures, transfixiante. Son état est nauséeux B une prise de corticoïdes sans vomissement. Elle est enceinte de 8 mois ; la gros- C un traitement antihistaminique sesse s'est déroulée normalement jusqu'à présent ; elle D l'installation d'une hypothyroïdie ne prend aucun traitement. Elle est apyrétique ; la BU E une insuffisance cardiaque est normale. L'enfant est viable selon les obstétriciens. Question 4 L'examen est pauvre en dehors d'un ventre doulou- Sur le plan thérapeutique, quelle(s) mesure(s) sont reux à la palpation, d'une tachycardie à 100 avec nécessaire(s) ? une PA conservée 125/75 mm Hg. Son poids physio- A envisager un accouchement provoqué ou une césarienne logique est à 78  kg pour 1,75  m et elle a pris 6  kg B mettre en place immédiatement une diète hydrique pendant sa grossesse. sans lipides Il existe comme antécédents familiaux une notion de C discuter la réalisation d'aphérèses en urgence diabète de type 2 chez son père âgé de 58 ans. L'échographie effectuée par l'obstétricien de garde D mettre en place une statine compte tenu de l'aug- mentation de sa cholestérolémie totale est normale ; il n'y a aucune métrorragie ou leucor- E par la suite mettre en place une nutrition compor- rhée. Son ECG est normal. tant moins de 20 g de lipides par jour Le laboratoire appelle en signalant que sur son bilan d'entrée fait à jeun le plasma est lactescent. Vous la Question 5 réinterrogez et apprenez que la veille au soir il y avait eu un repas festif mais sans consommation d'alcool Dans le cadre des mesures diététiques ultérieures, les du fait de la grossesse. aliments suivants sont déconseillés : A huile à base de triglycérides à chaînes moyennes Question 1 B assaisonnement avec huile de colza Dans cette situation, vous évoquez en priorité : C blanc de poulet sans peau A un infarctus du myocarde inférieur D lait demi-écrémé B une occlusion aiguë du grêle E pain (baguette, flûte) 460 C une colique néphrétique Question 6 D une pancréatite aiguë hypertriglycéridémique Par la suite, le séquençage revient positif avec une muta- E une pyélonéphrite aiguë tion faux sens homozygote de la LPL considérée comme Question 2 classiquement délétère. Finalement, la fin de grossesse Le dosage des amylases revient positif à 1 200 UI (N se termine dans de bonnes conditions avec une pancréa- tite aiguë résolutive, une hypertriglycéridémie contenue < 40) et un contrôle ciblé de l'échographie abdomi- sous mesures diététiques strictes autour de 12 g/l et un nale ne permet pas de visualiser le pancréas et ne voit accouchement avancé. Son mari qui a une discrète obé- pas d'épanchement intra-abdominal. sité abdominale a une triglycéridémie normale haute. Le bilan lipidique est complété : À long terme, la malade est exposée à un surrisque : • triglycérides : 75 g/l (N = 0,7-1,7) ; A de récidive de pancréatite aiguë • cholestérol total : 6 g/l (N = 1,5-2,2) ; B de cholécystite aiguë • ApoB : 1,2 g/l (N = 0,8-1,2) ; C d'infertilité • calcul du LDL-cholestérol impossible ; D de polynévrite • HDL-cholestérol : 0,28 g/l (N = 0,4-0,7). À l'interrogatoire, il y a une notion d'hypertriglycéridé- E de glomérulonéphrite chronique mie modérée chez son père, aucune anomalie lipidique Question 7 connue chez ses deux sœurs et sa mère a un bilan lipi- Elle vous interroge sur un risque de transmission de dique subnormal avec une triglycéridémie à 2 g/l sans son hypertriglycéridémie à son enfant. hypercholestérolémie. Une contraception œstropro- Quelle(s) est (sont) la (les) proposition(s) exacte(s) ? A vous êtes rassurant car la pathologie est récessive et la gestative lui avait été contre-indiquée il y a une dizaine probabilité que son mari soit hétérozygote est faible d'années car une hypertriglycéridémie franche avait B si jamais son mari était hétérozygote, il y aurait une été détectée sous contraception orale minidosée. chance sur quatre pour que son enfant soit affecté Vous évoquez au plan phénotypique : par une hyperchylomicronémie familiale A une hyperlipidémie de type I ou V C si jamais son mari était hétérozygote, il y aurait une B une hyperlipidémie de type IIb C une hyperlipidémie de type III chance sur deux pour que son enfant soit affecté D une hyperlipidémie de type IIa E anomalie inclassable par une hyperchylomicronémie familiale D si jamais son mari était hétérozygote, il y aurait une Question 3 hypertriglycéridémie sévère Le plasma décanté reste lactescent mais avec une E elle pourrait avoir des enfants sans surrisque d'hy- couche crémeuse évoquant bien une hyperlipidémie pertriglycéridémie modérée si son mari n'avait pas de type V, ce qui correspond aux paramètres lipidiques. de mutation de la LPL

Cas cliniques 25 Question 8 Question 13 461 Elle vous adresse pour avis son père, qui présente un Effectivement, il présente une très probable NASH diabète de type 2, une obésité abdominale modérée avec des ALAT à 95 UI (N < 45) et des ASAT à 52 UI Entraînement (IMC 31  kg/m2, tour de taille 106  cm). Sa triglycéri- (N < 45) avec un foie homogène hyper-réfringent en démie oscille depuis plusieurs années entre 3 et 6 g/l. échographie. Vous complétez son bilan et découvrez une microal- L'activité physique régulière dans sa situation devrait : buminurie, une rétinopathie non proliférante minime A faire baisser la triglycéridémie et vous avez la notion d'un athérome carotidien bila- B faire baisser les HDL-cholestérol téral sans sténose serrée. Son traitement comporte de C faire baisser le LDL-cholestérol la metformine 2 g par jour en monothérapie et 5 mg D améliorer son contrôle tensionnel de ramipril. Son HbA1c est à 7,3 %. E améliorer ses transaminases Son dernier bilan lipidique retrouve : • triglycérides : 4,8 g/l (N = 0,7-1,7) ; Question 14 • LDL-cholestérol mesuré : 0,8 g/l (N = 0,7-1,6) ; Quelques mois plus tard, alors qu'il est traité par féno- • HDL-cholestérol : 0,28 g/l (N = 0,4-0,7). fibrate, metformine, gliptine, ramipril 5 mg et que sa Quelles adaptations thérapeutiques proposez-vous au triglycéridémie est revenue à 2 g/l, il se plaint de myal- plan lipidique ? gies chroniques. A continuer avec les mesures diététiques en place Quelle(s) est (sont) la (les) proposition(s) exacte(s) ? B introduire une statine A vous vérifiez ses CPK C introduire un fibrate B vous suspendez le fibrate D introduire une association statine-fibrate C vous remplacez le fibrate par de l'Ezetrol® E introduire de l'ézétimibe D vous remplacez le fibrate par des acides gras n-3 E vous employez une statine à forte dose Question 9 Vous considérez que son hypertriglycéridémie : Question 15 A est peu sensible aux mesures diététiques Il vous apprend que sa fille envisage désormais une B est probablement plus sensible à l'apport en graisse seconde grossesse. Vous considérez : A qu'elle est formellement contre-indiquée qu'en glucides B qu'elle est envisageable sous réserve d'une surveil- C s'améliorera partiellement avec un meilleur contrôle lance rapprochée de la triglycéridémie glycémique C qu'elle est envisageable en suspendant transitoire- D s'améliorera partiellement en accroissant son acti- ment le régime hypolipidique pendant la grossesse vité physique quotidienne D qu'il est indispensable de mettre en œuvre une E est aggravée par sa microalbuminurie complémentation en acides gras oméga 3 pendant Question 10 la grossesse Il exerce la profession de chauffeur de taxi. Quelle(s) E qu'elle devra être suivie par un centre spécialisé en adaptation(s) thérapeutique(s) proposez-vous au plan lipidologie et une maternité de niveau 3 glycémique ? A associer un sulfamide Cas clinique 18 B associer une gliptine C associer un agoniste du GLP-1 Un homme de 47 ans, carrossier, est admis pour une D introduire une insulinothérapie nocturne hyperglycémie (2,20 g/l soit 12,1 mmol/l) découverte E mettre en place un schéma insulinique basal-bolus dans un contexte de perte de poids et de syndrome polyuro-polydipsique récent. Il a en effet perdu 4 kg Question 11 ces 3 derniers mois. Il ne se plaint pas de troubles Il vous interroge sur la consommation d'aliments sus- digestifs, ni d'altération de l'état général. ceptibles d'accentuer son hypertriglycéridémie. Vous On note dans ses antécédents personnels : lui conseillez : • une hypertension artérielle traitée par énalapril A d'employer des margarines enrichies en phytostérols B de limiter son apport de glucides à moins de 50 % (Renitec®) 5 mg, 1 cp. par jour depuis 2 ans ; C de réduire la consommation de fruits à deux unités • un tabagisme de 20 paquets-années ; • une consommation quotidienne de 1  litre de vin quotidiennes lors des repas D de remplacer les sodas par des boissons au fructose + deux à trois apéritifs. E de suivre un régime globalement hypocalorique On note dans ses antécédents familiaux : • sa mère, obèse, souffrait de diabète apparu à la Question 12 Dans cette situation métabolique, il est fréquent de cinquantaine, longtemps équilibré par des médica- rencontrer : ments hypoglycémiants oraux et mise à l'insuline à A une hyperuricémie 67 ans ; elle est actuellement traitée pour un can- B une stéatose hépatique cer du côlon ; C une hépatite stéatosique • sa grand-mère maternelle était aussi obèse ; elle D une hypertension artérielle est décédée d'un cancer du rectum ; E une dysautonomie neurovégétative • son père a présenté une thyroïdite de Hashimoto et est traité par la lévothyroxine.

Cas cliniques Examen à l'admission : Question 5 • poids : 90 kg pour 1,67 m ; tour de taille : 95 cm ; Quel(s) examen(s) complémentaire(s) envisagez-vous circonférence au niveau des hanches : 87 cm ; de demander en première intention ? • PA : 135/80 mm Hg sous traitement hypotenseur ; A un typage HLA • examen clinique sans particularité si ce n'est un B une échographie pancréatique par voie endogas- discret débord hépatique ; trique • glycémie : 1,80 g/l [9,9 mmol/l], alors qu'il fait un C un scanner pancréatique régime depuis 3 semaines ; D une recherche d'anticorps anti-GAD • cholestérol total : 2,35 g/l [6,08 mmol/l] ; E un bilan martial (sidérémie, coefficient de satura- • triglycérides : 2,45 g/l [2,77 mmol/l] ; tion de la sidérophiline, ferritinémie) • HDL-cholestérol : 0,35 g/l [0,9 mmol/l] ; Question 6 • γ-GT : 65 UI/l ; • ALAT : 90 ; ASAT : 105 UI ; Quel(s) médicament(s) hypoglycémiant(s) est (sont) • NFS : GR : 4,5 T/l ; GB : 6,5 G/l ; Hb : 13 g/l ; He : susceptible(s) de déclencher des troubles digestifs 47 % ; VGM : 104 μm3 ; plaquettes : 250 G/l ; réti- culocytes : 20 G/l. (perte d'appétit, diarrhée) ? A metformine B sulfamides Question 1 C analogues du GLP-1 Pour poser définitivement le diagnostic de diabète : D glinides A vous devez revérifier la glycémie à jeun E analogues lents de l'insuline B vous devez réaliser une épreuve d'hyperglycémie Question 7 provoquée par voie orale La recherche des anticorps anti-GAD s'est révélée C vous devez réaliser un cycle glycémique négative. Le bilan martial est normal. Le bilan thyroï- D vous devez demander une mesure de l'HbA1c dien est normal. Le patient va passer un scanner du E vous n'avez pas à compléter les investigations pancréas. Question 2 Quand les anticorps anti-GAD sont négatifs et que Quel(s) est (sont) l'(les) élément(s) de l'observa- vous soupçonnez très fortement un diabète de tion qui orienterai(en)t plutôt vers un diabète de type  1, quel(s) autre(s) autoanticorps peut (peuvent) type 2 ? 462 A l'obésité avec un morphotype abdominal être recherché(s) ? A anti-peptide C B l'existence d'un syndrome métabolique B IA-2 C le tabagisme actif C anti-insuline D la consommation excessive de boissons alcoolisées D anti-canaux calciques ATP-dépendants E les antécédents familiaux de diabète E anti-ZnT8 Question 3 Question 8 Quel médicament antidiabétique envisagez-vous de Quelle(s) maladie(s) pancréatique(s) peut (peuvent) prescrire chez ce patient ? être responsable(s) d'un diabète secondaire ? A sulfamide A pancréatite chronique B metformine B cancer du pancréas C inhibiteurs des DPP-4 C insulinome D analogues du GLP-1 D mucoviscidose E analogue lent de l'insuline glargine au coucher E pancréas divisum selon un schéma dit « bed time » Question 9 Le scanner montre de nombreuses calcifications au Question 4 niveau d'un canal pancréatique principal (canal de Vous le revoyez 4 mois plus tard. Wirsung) dilaté et permet de poser le diagnostic de Le patient avait été mis sous metformine 1000, 1 cp. pancréatite chronique calcifiante. L'introduction de 3  fois par jour. Son équilibre s'est amélioré mais sulfamides hypoglycémiants (glibenclamide 5  mg demeure insuffisant et son HbA1c est à 7,3  %. Il a 3 fois par jour) n'a pas permis par ailleurs de stabiliser encore perdu 3 kg alors que l'observance du régime l'HbA1c (7,2 %). paraît médiocre. Il se plaint de troubles digestifs (perte Quelle adaptation du traitement antidiabétique allez- d'appétit, selles molles). vous mettre en place ? Quel(s) type(s) de diabète autre(s) que le diabète A augmenter les doses de metformine de type  2 pourrai(en)t expliquer ce résultat clinique B introduire un analogue du GLP-1 insuffisant ? C associer un inhibiteur des glucosidases digestives à A un diabète de type 1 lent (LADA) B un diabète secondaire pancréatique la metformine et au sulfamide C un diabète secondaire à une hypothyroïdie D mettre en place une insulinothérapie « bed time » D un diabète iatrogénique E un diabète monogénique associée à la metformine E mettre en place une insulinothérapie intensifiée

Cas cliniques 25 Question 10 Par rapport au diabète de type 1, le diabète associé à 463 Le diabète survenant au cours de la pancréatite chro- la pancréatite chronique est caractérisé par : nique calcifiante est caractérisé par : A le risque d'hypoglycémie est augmenté dans le dia- Entraînement A son apparition très précocement dans l'histoire bète de la pancréatite chronique traité par insuline naturelle de la maladie par rapport à ce qu'il est dans le diabète de type 1 B son apparition favorisée par les prises en charge B les doses d'insuline nécessaires au bon contrôle du diabète sont plus faibles dans la pancréatite chro- chirurgicales des complications de la maladie nique que dans le diabète de type 1 C son retentissement aussi sur les cellules alpha des C la cinétique des insulines est raccourcie dans la pan- créatite chronique par rapport au diabète de type 1 îlots de Langerhans, provoquant une diminution D la sécrétion de glucagon est altérée dans la pancréa- de la sécrétion de glucagon tite chronique et augmentée dans le diabète de type 1 D la persistance d'une certaine sécrétion insulinique E les conséquences nutritionnelles de la pancréa- résiduelle, ayant de ce fait un risque cétogène tite chronique peuvent retentir sur la stabilité du moindre que dans d'autres diabètes insulinoprives diabète E une insulinorésistance primitive des tissus cibles de l'insuline : foie, muscle et tissu adipeux Question 15 Quelle est la définition internationale des malaises Question 11 hypoglycémiques sévères ? Quelle(s) va (vont) être votre (vos) décision(s) pour la A une glycémie inférieure à 0,20 g/l prise en charge de la pancréatite chronique ? B une glycémie inférieure à 0,70 g/l A régime hypocalorique et délipidé C un malaise hypoglycémique qui nécessite la partici- B arrêt des boissons alcoolisées C prescription d'extraits pancréatiques pation d'un tiers pour être reconnue et/ou traitée D supplémentation en vitamine D D une hypoglycémie qui se traduit par des symp- E duodénopancréatectomie tômes constrictifs thoraciques Question 12 E une hypoglycémie nocturne Le patient est actuellement traité par insuline : ana- logue rapide Aspart (Novorapid®) 5 U le matin, 7 U le Cas clinique 19 midi et 6 U le soir, et analogue lent glargine Lantus®, 16 U le soir. Il prend des extraits pancréatiques Créon® Une femme de 58 ans est amenée aux urgences par 25000, 1 cp. à chaque repas. Il ne boit plus d'alcool. Il sa fille qui l'a trouvée le matin chez elle difficilement a stabilisé son poids à 88 kg. L'HbA1c est à 5,8 %. Il réveillable et confuse après le réveil. La fille rapporte présente de fréquents malaises (vertiges, faim doulou- que depuis 2 mois sa mère était fatiguée, buvait beau- reuse, confusion) accompagnés de sueurs survenant à coup d'eau (4–5  litres) et  allait beaucoup uriner et distance des repas. Il a même perdu connaissance et a n'avait plus d'appétit. Elle est suivie par son médecin convulsé à deux reprises en fin de nuit. traitent pour une hypertension artérielle traitée par Quelle est la cause la plus plausible des malaises qu'il ramipril 5 mg par jour et a des antécédents de cancer présente ? [Une seule réponse.] du sein traité par chirurgie il y a 4  ans. À l'examen A une insuffisance de la dose de l'extrait pancréa- clinique, la patiente est somnolente et asthénique, sans troubles neurologiques ; elle a une langue sèche. tique (Créon®) B une reprise de l'alcoolisme Question 1 C des hypoglycémies Quel(s) examen(s) biologique(s) permettra (permet- D des poussées de pancréatite aiguë tront) d'avancer dans le diagnostic différentiel du E une hypotension orthostatique dans le cadre de syndrome polyuro-polydipsique chez cette patiente ? A la glycémie son traitement hypotenseur B la calcémie C la kaliémie Question 13 D l'osmolarité urinaire Compte tenu du contexte clinique, quel est votre sen- E les bicarbonates plasmatiques timent sur la valeur actuelle d'HbA1c ? A elle se situe dans les objectifs thérapeutiques Question 2 B elle est encore trop élevée Vous recevez le bilan biologique suivant : C elle est trop basse • NFS : GB : 5,1 · 109/l ; hémoglobine : 15 g/l ; héma- D elle est ininterprétable du fait du risque de cirrhose tocrite : 47 % ; plaquettes : 120 · 109/l ; du foie sur ce terrain • glycémie : 4,2 mmol/l ; E je n'en tiens pas compte car il est préférable • natrémie : 142 mmol/l ; kaliémie : 4,2 mmol/l ; cal- de se fonder sur les glycémies pour évaluer cémie : 3,70 mmol/l ; l'équilibre glycémique dans le cas des diabètes • créatinine plasmatique : 112 μmol/l ; urée plasma- secondaires tique : 15,9 mmol/l ; Question 14 • protidémie : 80 g/l. Il s'agit bien d'hypoglycémies puisque des glycémies autour de 0,20–0,40 g/l ont été constatées et que les malaises régressent après charge en sucre.

Cas cliniques Ce bilan : • calciurie des 24 heures : 14 mmol par 24 heures (N A confirme un diabète sucré pour une femme < 6,0) ; B confirme un diabète insipide • PTH : 112 pg/ml (N = 15-60). C confirme une hypercalcémie Vous concluez donc à une hyperparathyroïdie primaire. D montre une fausse hypercalcémie liée à l'hyperpro- Le taux de 25-(OH)-vitamine D est à 15 ng/ml (N > 30). tidémie À ce stade, la substitution du déficit en vitamine D par E montre une insuffisance rénale fonctionnelle Uvedose® 1 ampoule par mois chez cette patiente : Question 3 A est recommandée Vous avez diagnostiqué chez cette patiente une B est contre-indiquée car elle peut induire une crise hypercalcémie maligne. Vous prenez en charge la patiente aux urgences et vous mettez en place le (les) hypercalcémique traitement(s) suivant(s) : C peut diminuer le taux de la PTH A une perfusion de sérum glucosé G5, 3  litres lors D peut augmenter le taux de la PTH E permet de prévenir le syndrome des os avides de des 12 premières heures calcium (hungry bone syndrome) B une perfusion de sérum physiologique à 0,9  %, Question 8 3 litres lors des 12 premières heures Quel(s) examen(s) morphologique(s) sont pertinents C une perfusion de sérum physiologique à 0,9  %, pour rechercher les retentissements potentiels de 1 litre lors des 12 premières heures cette hyperparathyroïdie ? D furosémide 40 mg IVL A scanner rénal E méthylprednisolone 500 mg IV B échographie thyroïdienne Question 4 C ostéodensitométrie Après la prise en charge initiale aux urgences et en réani­ D radiographie des genoux mation, l'état de la patiente se stabilise. Elle est hospi- E scintigraphie parathyroïdienne talisée dans le service d'endocrinologie pour poursuivre Question 9 le diagnostic et le traitement de cette hypercalcémie. L'hyperparathyroïdie primaire prédispose à l'ostéopo- Vu le contexte clinique, quelles sont, selon vous, les rose. Quelle est l'atteinte osseuse préférentielle, typique deux causes d'hypercalcémie les plus probables chez dans cette maladie, liée à l'action de la PTH dans l'os ? 464 cette patiente ? A ostéoporose du radius A hyperparathyroïdie primaire B ostéoporose du col fémoral B hypercalcémie hypocalciurique familiale C ostéoporose du rachis lombaire C myélome multiple D ostéoporose de l'os trabéculaire D hypercalcémie par sécrétion paranéoplasique de la E ostéoporose de l'os cortical PTHrp par le cancer du sein Question 10 E sarcoïdose Une prise en charge chirurgicale est décidée chez Question 5 cette patiente puisqu'elle présentait une hypercalcé- Parmi les examens biologiques suivants, un seul est mie sévère avec une hypercalciurie. En plus, le scanner essentiel au diagnostic étiologique initial de l'hyper- rénal a identifié une néphrolithiase. calcémie : lequel ? Quel(s) examen(s) de localisation demandez-vous A calciurie de 24 heures avant la chirurgie parathyroïdienne en première B vitamine D intention ? C calcitriol A une échographie parathyroïdienne D PTH B un scanner parathyroïdien E PTHrp C une IRM cervicale Question 6 D une scintigraphie au Tc-Sesta MIBI Il est indispensable pour avancer dans le diagnostic E une scintigraphie à l'iode 131 étiologique de doser la PTH. Quelles sont en général Question 11 les causes d'hypercalcémie avec un taux de PTH nor- Avant l'opération, la patiente avoue avoir eu des mal ou élevé (hypercalcémie PTH-dépendante) ? coliques néphrétiques depuis l'âge de 28 ans, jamais A hyperparathyroïdie primaire B hyperparathyroïdie secondaire traitées. Cette nouvelle information : C hyperparathyroïdie tertiaire A fait suspecter une cause génétique de l'hyperpara- D hypercalcémie hypocalciurique familiale E déficit en vitamine D thyroïdie primaire chez cette patiente B indique l'analyse génétique du gène MEN1 (codant la ménine) Question 7 C indique l'analyse génétique du gène HRPT2 Après l'hyperhydratation, le bilan phosphocalcique de la patiente est le suivant : (codant la parafibromine) • calcémie : 2,75 mmol/l ; D indique l'analyse génétique du gène CaSR (codant • phosphatémie : 0,68 mmol/l ; le récepteur sensible au calcium) E oriente le caractère du geste opératoire

Cas cliniques 25 Question 12 C échographie thyroïdienne 465 Vous apprenez également que la patiente souffre, D scintigraphie thyroïdienne depuis plusieurs années, de diarrhées chroniques et que E scanner cervical Entraînement son oncle paternel est mort d'une hémorragie digestive. Dans ce contexte, les diarrhées font suspecter : Question 4 A un gastrinome Quel(s) diagnostic(s) étiologique(s) suspectez-vous ? B un insulinome A une maladie de Basedow C un phéochromocytome B une thyroïdite de Hashimoto D un prolactinome C une thyroïdite de De Quervain E un VIPome D une thyroïdite aiguë E un cancer anaplasique de la thyroïde Cas clinique 20 Question 5 Madame T., âgée de 29  ans, a pris 4  kg en un an. Quel(s) examen(s) biologique(s) va (vont) vous per- Elle vous consulte en raison d'une perte de cheveux mettre de porter le diagnostic de thyroïdite lympho- « affreuse et affolante » et de la découverte fortuite, à cytaire chronique ? l'autopalpation cervicale, d'un nodule thyroïdien. Ses A dosage des anticorps anti-thyroperoxydase (anti- principaux antécédents consistent en un vitiligo et un diabète insulinodépendant depuis 12 ans. Elle se fait TPO) à des taux élevés quatre injections d'insuline par jour et est parfaitement B dosage des anticorps anti-récepteurs de la TSH à équilibrée. Elle est traitée par un IEC pour une HTA à prédominance diastolique. Elle est la mère d'une petite des taux élevés fille de 6 ans venue à terme, par voie naturelle, sans C VS accélérée macrosomie, qui fait la joie de sa maman. Cette der- D CRP augmentée nière accuse toutefois la fatigue de s'occuper d'une E présence d'anticorps antinucléaires de type fillette très « vivante » et présenterait depuis quelques mois une constipation inhabituelle. Elle est également moucheté traitée par fibrates pour une dyslipidémie. À l'examen cervical, le nodule est situé à la base du Question 6 lobe droit et mesure environ 2 cm. Il est ferme, indo- En fin de consultation, la patiente vous interpelle lore, situé au sein d'un goitre nodulaire plutôt dur. parce que, depuis environ 2  mois, elle souffre de Vous découvrez par ailleurs un second nodule à la crampes des membres inférieurs, qui ne s'améliorent base du lobe gauche, mesurant environ 1 cm, ferme à pas malgré l'arrêt de toute pratique sportive. dur, et un troisième nodule au sommet du lobe droit, Quelles sont vos hypothèses diagnostiques ? dur, de 2,5  cm environ. L'examen clinique révèle en A souffrance musculaire secondaire à une hypothyroïdie outre une xérodermie et une dépilation de la queue B toxicité des hypolipémiants des sourcils. C phlébite surale bilatérale du myxœdémateux D pseudopolyarthrite rhizomélique Question 1 E maladie de Horton Quels sont les symptômes qui vont vous orienter vers une hypothyroïdie ? Question 7 A asthénie Les résultats biologiques vous parviennent au bout de B nodule thyroïdien quelques heures. La TSH est dosée à 25  mU/l (VR  : C HTA à prédominance diastolique 0,4-4) ; les CPK sont à 12 fois la normale. D troubles des phanères Quelle est votre attitude thérapeutique ? E xérodermie A arrêt des fibrates B prescription d'une statine Question 2 C introduction de carbimazole Quel(s) examen(s) biologique(s) demandez-vous en D introduction de lévothyroxine première intention ? E mise en route d'une contraception A T3 B TSH Question 8 C TRAK Concernant les modalités de prise en charge chez D thyroglobuline votre patiente, quelle(s) est (sont) la (les) proposition(s) E anticorps anti-thyroglobuline exacte(s) ? A hospitalisation pour la mise en route de la Question 3 Quel(s) examen(s) non biologique(s) demandez-vous lévothyroxine en première intention ? B introduction progressive de la lévothyroxine A examen ophtalmologique C dose de charge de lévothyroxine puis décroissance B IRM hypophysaire jusqu'à la posologie minimale efficace D corticothérapie associée à 0,5 mg/kg E répartition de la lévothyroxine en 40  % pour la prise du matin, 40 % pour la prise de midi et 20 % pour la prise du soir

Cas cliniques Question 9 Question 14 Quel(s) est (sont) le(s) paramètre(s) pertinent(s) pour la Votre patiente accouche à terme. À sa sortie de la surveillance au long cours de votre patiente ? maternité, cette jeune maman vous interroge  : A dosage de la T3 « Sachant que je suis sous Lévothyrox®, quel est le B dosage de la T4 risque d'allaiter ? Ce traitement peut-il jouer sur la C dosage de la TSH \"qualité\" de mon lait ? » D dosage des anticorps anti-TPO Quelle(s) est (sont) la (les) proposition(s) exacte(s) ? E dosage des anticorps anti-thyroglobuline A vous lui répondez que l'allaitement est formelle- Question 10 ment contre-indiqué B vous lui répondez que l'allaitement peut être pro- Après avoir atteint la posologie cible de lévothyroxine, posé sans restriction quand contrôlez-vous la TSH pour la première fois ? C vous lui répondez que l'allaitement doit être A après 1 jour B après 1 semaine évité dans les 2  heures qui suivent la prise de la C après 1 mois D après 3 mois lévothyroxine E après 1 an D vous lui répondez que l'allaitement doit être évité dans Question 11 les 2 heures qui précèdent la prise de la lévothyroxine E vous lui répondez qu'il vaut mieux tirer le lait et le La TSH, lors d'un contrôle effectué 3 mois plus tard, laisser décanter 12  heures au réfrigérateur avant est à 1,11 mU/l ; la T4L est au-dessus des normes de utilisation 466 5  % ; les CPK se sont normalisées tout comme les Question 15 paramètres lipidiques. Elle vous consulte à nouveau environ 4 mois après son Interprétez ces résultats et commentez la suite de accouchement eutocique pour une asthénie. Le retour votre prise en charge. Quelle(s) est (sont) la (les) de couche a été rapide. L'examen clinique est pauvre. proposition(s) exacte(s) ? La patiente dit éprouver une grande lassitude à chaque A il faut réintroduire le fibrate maintenant que la fois qu'elle doit « changer [son] petit Adalbert ». fonction thyroïdienne s'est normalisée B il faut réduire la lévothyroxine de 12,5 μg par jour Quels sont les diagnostics que vous évoquez ? C on peut arrêter la supplémentation thyroïdienne A « baby blues » D la fonction thyroïdienne s'inscrit en euthyroïdie B dépression du post-partum E il est nécessaire de doser la T3 C thyroïdite du post-partum D déséquilibre de la substitution thyroïdienne E rhabdomyolyse secondaire à la prise de statines Question 12 Question 16 Un an plus tard, la patiente est enceinte et vous Un an plus tard, la patiente, toujours sous traitement, consulte, affolée, car elle a peur de malformations vous consulte pour une aménorrhée. Elle décrit des fœtales du fait de sa dysthyroïdie. En quoi la gestation bouffées de chaleur et une dyspareunie. Il n'y a pas de peut-elle affecter l'équilibre thyroïdien ? mastodynies. Le test de grossesse est négatif. La patiente A la grossesse entraîne la production d'une hormone revient 10 jours plus tard avec les résultats suivants : susceptible de stimuler la thyroïde de la femme • prolactine : 18 ng/ml (N = 2-25) ; enceinte B la grossesse entraîne la production d'une hormone • FSH : 188 UI/l ; inhibitrice de la thyroïde de la femme enceinte • LH : 12 UI/l ; C la grossesse entraîne une majoration de la pro- • œstradiol : 8 μg/l (valeur très basse) ; duction de TBG, protéine vectrice des hormones • testostéronémie libre : 0,4 ng/ml (VR : 0,4-0,6) ; thyroïdiennes • TSH : 1,01 mU/l. D la grossesse s'accompagne d'une majoration Comment interprétez-vous l'aménorrhée au vu de ces des besoins en hormones thyroïdiennes chez la résultats ? patiente substituée A grossesse extra-utérine E la grossesse s'accompagne d'une diminution B hypogonadisme hypogonadotrophinique C adénome à prolactine des besoins en hormones thyroïdiennes chez la D hyperandrogénie E ménopause patiente substituée Question 13 Question 17 Parmi ces différents facteurs, lequel (lesquels) est En définitive, vous évoquez une atteinte syndromique (sont) potentiellement goitrigène(s) chez la femme chez votre patiente. Laquelle (lesquelles) est (sont) enceinte ? le(s) plus plausible(s) ? A la lévothyroxine prise par la patiente A diabète lipoatrophique B la majoration de la production de TBG B hyperprolactinémie de déconnexion C la carence iodée C néoplasie endocrinienne multiple de type 1 D l'hCG D néoplasie endocrinienne multiple de type 2 E aucun de ces paramètres E polyendocrinopathie auto-immune

Cas cliniques 25 Cas clinique 21 C OsmP = 2 × (Natrémie + Kaliémie) + Glycémie en 467 mmol/l + Protidémie en mmol/l Madame M., 78 ans, mère de trois enfants, ménopau- Entraînement sée à 44 ans, autonome, est amenée aux urgences à D OsmP = 2 × Natrémie + Glycémie en mmol/l – Tri- la suite d'une chute inexpliquée sur la voie publique glycérides en mmol/l sans perte de connaissance ni traumatisme crânien, alors qu'elle attendait pour traverser. Elle ne reçoit E l'alcool augmente l'osmolarité plasmatique de pas d'autres médicaments que du paracétamol en rai- 22 mOsmol/g d'alcool son de maux de tête assez fréquents attribués à une arthrose cervicale. Question 5 Elle présente à l'entrée une confusion. La PA est Devant ce tableau de syndrome de sécrétion inap- à 110/60  mm  Hg. Le poids est de 59  kg. Il existe proprié d'hormone antidiurétique (SIADH), quel(s) des excoriations des deux genoux. Il n'y a pas de dosage(s) hormonal(aux) devez-vous effectuer de façon signes de focalisation neurologique, ni de signes cli- systématique ? niques de déshydratation ou d'inflation hydrosodée. A TSH et T4L Une fracture de la fibula sans gravité est mise en B TSH seule évidence. C cortisolémie La biologie montre : D FSH • sérum non lactescent ; E prolactine • glycémie : 0,69 g/l ; • natrémie : 115 mmol/l ; Question 6 • kaliémie : 4,4 mmol/l ; Parmi les propositions suivantes, citez la (les) cause(s) • urée : 0,10 g/l ; possible(s) de SIADH ? • créatininémie : 6 mg/l ; A antidépresseurs • protidémie : 60 g/l. B cancer à petites cellules C pneumocystose pulmonaire Question 1 D accident vasculaire cérébral Parmi les propositions suivantes concernant le tableau E fracture fémorale clinique, indiquez celle(s) qui est (sont) attendue(s) chez cette patiente : Question 7 A un pli cutané Parmi les propositions suivantes concernant le tableau B des œdèmes des membres inférieurs clinico-biologique de cette patiente, quel(s) élément(s) C une sécheresse des muqueuses n'est (ne sont) pas en faveur d'une insuffisance primi- D des troubles de l'équilibre tivement surrénalienne ? E des nausées A l'hyponatrémie B la tendance hypoglycémique Question 2 C l'urée basse Parmi les propositions suivantes concernant le tableau D la normokaliémie biologique, indiquez celle(s) qui oriente(nt) vers une E l'absence de signes cliniques de déshydratation hypo-osmolalité chez cette patiente : A l'absence d'hyperprotidémie extracellulaire B la normokaliémie C l'urée basse Question 8 D l'absence d'hyperglycémie La cortisolémie prélevée à l'arrivée de la patiente à E l'absence de lactescence du sérum 17 h est inférieure à 1 μg/dl ; la TSH est normale ; la T4L est à 4 pmol/l (N = 9-16). Question 3 Quelle(s) est (sont) la (les) proposition(s) exacte(s) ? Parmi les propositions suivantes concernant cette A panhypopituitarisme (antérieur et postérieur) patiente, indiquez laquelle (lesquelles) caractérise(nt) B insuffisance thyréotrope son tableau biologique : C insuffisance corticotrope A hyponatrémie hypo-osmolaire normovolémique D la FSH est attendue élevée B hyponatrémie hypovolémique E la vasopressine sera indosable C syndrome hyperosmolaire hypoglycémique D déshydratation extracellulaire Question 9 E hyperhydratation intracellulaire Devant ce tableau d'hyponatrémie avec confusion, quelle conduite à tenir préconisez-vous aux urgences ? Question 4 A 1 litre de sérum glucosé isotonique par 12 heures Parmi les propositions suivantes concernant l'osmola- rité plasmatique (OsmP), indiquez laquelle (lesquelles) en attendant les résultats hormonaux est (sont) exacte(s) : B restriction hydrique à 100 cc/24 heures A les produits de contraste radiologiques augmen- C sérum salé hypertonique en seringue autopulsée tent l'osmolarité plasmatique jusqu'à correction de la natrémie à 140 mmol/l B OsmP = 2 × (Natrémie + Kaliémie) + Glycémie en D diurétiques E restriction hydrosodée mmol/l + Urée en mmol/l Question 10 L'hypopituitarisme antérieur est confirmé. La patiente a retrouvé ses esprits sous hydrocortisone 20  mg

Cas cliniques par jour et lévothyroxine 75 μg par jour. La natrémie s'est normalisée et les céphalées ont disparu. La FSH, l'IGF-1 et la prolactinémie sont basses. L'IRM hypo- physaire est la suivante. Question 1 Quelle est la probabilité qu'un diabète sucré de type 2 soit secondaire à un syndrome de Cushing ? A 50 % B 30 % C 20 % Quelle attitude envisagez-vous ? D 5 % A résection chirurgicale par voie haute E 0 % 468 B résection chirurgicale par voie trans-sphénoïdale Question 2 C surveillance morphologique s'il n'y a pas de Dans l'hypothèse d'un syndrome de Cushing, quel(s) élément(s) clinique(s) le(s) plus spécifique(s) allez-vous troubles visuels rechercher ? D traitement par hormone de croissance A hirsutisme E la proximité des voies optiques impose une chirurgie Question 11 B amyotrophie quadricipitale Ce champ visuel est normal. C vergetures pâles Quelle attitude thérapeutique envisagez-vous ? D peau ecchymotique A résection chirurgicale par voie haute E obésité abdominale B résection chirurgicale par voie trans-sphénoïdale C surveillance morphologique Question 3 D surveillance ophtalmologique Parmi les symptômes présentés par la patiente, E traitement par hormone de croissance lequel (lesquels) peut (peuvent) être secondaire(s) à l'hypercortisolisme ? Cas clinique 22 A troubles psychiatriques B douleurs articulaires Une femme de 58 ans consulte pour une prise pon- C phlébite dérale de 7  kg survenue en 2  ans. Elle dit que son D psoriasis visage s'est arrondi en même temps. On lui a décou- E hypertension artérielle vert récemment lors d'une prise de sang une glycémie à jeun à 1,89 g/l. Enfin, elle contrôle sa PA à l'aide de Question 4 l'appareil d'automesure de son mari et a constaté à deux reprises un PA élevée à 170/110 mm Hg. Dans l'hypothèse d'un syndrome de Cushing, quel Dans ses antécédents, on retient un psoriasis, une phlébite surale survenue il y a 8  mois sans examen biologique de dépistage prescrivez-vous en circonstance particulière de survenue, et des troubles de l'humeur étiquetés psychose maniaco- première intention ? dépressive et traités depuis des années par de la A test de freinage minute à la dexaméthasone carbamazépine. B CLU des 24 heures C cortisol plasmatique à minuit D dosage de l'ACTH le matin à 8 h E test de freinage standard à la dexaméthasone

Cas cliniques 25 Question 5 Question 7 469 Parmi les situations suivantes, laquelle (lesquelles) La patiente a un syndrome de Cushing confirmé. Quel(s) peut (peuvent) être responsable(s) d'un accroissement examen(s) est (sont) à envisager en deuxième intention ? du CLU des 24 heures ? A IRM hypophysaire A hypothyroïdie B TDM surrénalienne B sevrage éthylique C coroscanner C surpoids D HGPO D dépression sévère E dosage de l'ACTH plasmatique E septicémie Question 8 Question 6 Son bilan est le suivant : Dans le cadre d'un syndrome de Cushing iatrogène, • CLU des 24 heures : 250 μg/24 heures (N < 80) ; quelle(s) est (sont) la (les) proposition(s) exacte(s) ? • Cortisol à minuit : 420 nmol/l (N < 200) ; A il ne survient qu'en cas de corticoïdes administrés • ACTH à minuit = 6 pmol/l (N < 2,2-5). Quelle est étiologie la plus probable du syndrome de par voie orale Cushing ? B il est caractérisé par une augmentation du CLU et A microadénome corticotrope B corticosurrénalome malin un taux d'ACTH plasmatique élevé C adénome cortisolique C il est caractérisé par une augmentation du CLU et D sécrétion ectopique d'ACTH E macroadénome corticotrope un taux d'ACTH plasmatique effondré D il est caractérisé par un CLU et un taux d'ACTH Question 9 Interprétez cette IRM. plasmatique effondrés E il expose au risque d'insuffisance surrénalienne périphérique lors du sevrage en corticoïdes Entraînement

Cas cliniques A  : avant injection ; B et C  : clichés précoces après Quelle(s) adaptations(s) de traitement est (sont) injection de produit de contraste ; D : cliché tardif. correcte(s) ? Quelle(s) est (sont) la (les) proposition(s) exacte(s) ? A diminuer le carbimazole A l'anomalie est compatible avec un microadénome B adjoindre de la L-thyroxine au carbimazole hypophysaire C arrêter le carbimazole B l'anomalie ne peut affirmer la maladie de Cushing D arrêter le propanolol introduit initialement C un test au CRH pourrait être utile pour préciser E donner une dose d'iode l'étiologie du syndrome de Cushing Question 4 D le test de freinage fort à la dexaméthasone est La patiente vous explique à la consultation suivante indispensable pour préciser l'étiologie du syn- qu'elle a présenté une angine il y a 15 jours. drome de Cushing E le cathétérisme des sinus pétreux inférieurs est Quelle(s) est (sont) la (les) proposition(s) juste(s) ? l'examen le plus performant pour préciser l'étiolo- Vous vérifiez qu'elle a bien eu un contrôle de la NFS et : A qu'il n'y a pas d'hyperlymphocytose gie du syndrome de Cushing B qu'il n'y a pas d'hyperleucocytose Question 10 C qu'il n'y a pas d'agranulocytose Concernant la maladie de Cushing, quelle(s) est (sont) D qu'il n'y a pas d'éosinophilie la (les) proposition(s) exacte(s) ? E qu'il n'y a pas de monocytose A elle peut s'accompagner d'un diabète insipide Question 5 B elle est responsable d'une surmortalité cardiovas- La NFS est normale. La patiente va bien mais elle culaire C elle peut entraîner un syndrome confusionnel trouve que ses selles sont molles. Elle a perdu un peu D elle peut s'accompagner de séquelles cognitives de poids ; la TSH est à 1,8 mUI/ml. et psychologiques même après une rémission Quelle(s) est (sont) la (les) cause(s) à évoquer dans ce complète E elle peut entraîner un retard de croissance statu- contexte de maladie de Basedow ? A récidive de l'hyperthyroïdie rale quand elle survient chez l'enfant prépubère B maladie cœliaque C parasitose D effet secondaire du carbimazole 470 Cas clinique 23 E colopathie fonctionnelle Une femme de 32  ans est adressée par son médecin Question 6 pour hyperthyroïdie. La TSH est freinée ; la T4L est à Étant donné le peu de recul, vous avez demandé à la 1,38 ng/dl (N = 0,75-1,46) ; la T3L est à 6,80 pmol/l (N patiente de vous rappeler si les selles molles persis- = 3,1-6,4). taient. Les troubles du transit ont rapidement disparu et étaient possiblement liés aux antibiotiques reçus Question 1 pour son angine. Quelle(s) est (sont) l'(les) étiologie(s) possible(s) de thy- Vous avez choisi un traitement associant carbimazole rotoxicose à T3L ? et L-thyroxine. La TSH de contrôle est à 5 mUI/ml. La A maladie de Basedow L-thyroxine est donnée à 50 μg. B thyroïdite de De Quervain Quelle(s) est (sont) la (les) proposition(s) juste(s) ? C adénome toxique A vous diminuez la L-thyroxine de 12,5 μg D hashitoxicose B vous diminuez la L-thyroxine de 25 μg E prise subreptice de L-thyroxine C vous augmentez la L-thyroxine de 25 μg Question 2 D vous augmentez la dose de 75 μg Vous demandez un bilan complémentaire. Quelle(s) E vous ne changez pas le traitement est (sont) la (les) proposition(s) juste(s) ? Question 7 A dosage de l'iodurie Malgré l'augmentation de la L-thyroxine, la TSH B dosage des anticorps anti-récepteurs de la TSH suivante reste élevée à 4,2  mUI. Vous pensez qu'il C dosage des anticorps anti-thyroglobuline y a peut-être quand même une maladie cœliaque D dosage pondéral des immunoglobulines associée. E dosage de la thyroglobuline Quelle(s) est (sont) la (les) proposition(s) juste(s) ? Question 3 A vous demandez un dosage des anticorps anti- Les anticorps anti-récepteurs de la TSH sont négatifs. transglutaminase L'échographie faite après la première consultation B vous demandez un dosage pondéral des immuno- objective une thyroïde un peu augmentée de volume, globulines hypervasculaire et contenant quelques micronodules C vous demandez un dosage des anticorps anti-­ bilatéraux : vous concluez à une maladie de Basedow. gliadine La patiente est sous carbimazole depuis la dernière D vous demandez un dosage des anticorps anti- consultation ; la T3L s'est normalisée, la T4L est dans mitochondies les valeurs basses de la norme. E vous demandez un dosage des anticorps anti-CCP

Cas cliniques 25 Question 8 Question 1 471 Le bilan biologique et la FOGD demandée ensuite Quel bilan réalisez-vous pour explorer ses troubles de pour biopsies duodénales sont normaux. Vous vérifiez la libido ? Entraînement les modalités de prise de la L-thyroxine. A dosage de la testostérone totale Quelle(s) est (sont) la (les) proposition(s) juste(s) ? B dosage des gonadotrophines plasmatiques (LH, FSH) A ne pas prendre de sels de fer de manière concomi- C dosage de la T4L D dosage du cortisol libre urinaire des 24 heures tante E dosage de la prolactine B prendre la L-thyroxine en mangeant C prendre la L-thyroxine au moins 20 minutes avant Question 2 Au cours de l'examen, vous êtes marqué par l'aspect de manger dysmorphique du patient et celui-ci vous précise qu'il D ne pas prendre de calcium de manière a gagné deux pointures et ne peut plus porter son alliance. Vous suspectez une acromégalie. concomitante Quelles complications de l'acromégalie devez-vous E prendre la L-thyroxine après une heure en position rechercher ? A un syndrome d'apnées du sommeil debout B une exophtalmie C un diabète Question 9 D une hypotension orthostatique La patiente demande à vous revoir plus rapide- E un syndrome du canal carpien ment en raison de problèmes oculaires. Elle a vu sur internet qu'il pouvait s'agir d'une orbitopathie Question 3 basedowienne. Quel(s) dosage(s) biologique(s) pouvez-vous prescrire Quelle(s) est (sont) la (les) proposition(s) juste(s) ? Pour pour confirmer le diagnostic d'acromégalie ? conforter ou infirmer l'orbitopathie, vous cherchez à A la GH plasmatique à minuit l'examen : B l'IGF-1 plasmatique A une rougeur conjonctivale C la GH lors d'un test d'hypoglycémie insulinique B une diplopie D la somatostatine plasmatique C des xanthélasmas E la prolactine plasmatique D un arc cornéen E une asynergie oculopalpébrale Question 4 L'acromégalie est confirmée. Question 10 Vous avez prescrit une IRM hypophysaire dont voici Il n'y a pas de signes d'orbitopathie. une image représentative. Vous la revoyez après un an de traitement. Vous avez fait contrôler l'échographie. Quelle(s) est (sont) la (les) proposition(s) juste(s) ? Vous vérifiez avant de prendre la décision d'arrêt du traitement : A que les micronodules ont disparu B que la thyroïde a retrouvé un volume normal C que l'hypervascularisation a disparu D que la thyroïde est hyperéchogène E que la thyroïde est hypovasculaire Cas clinique 24 Quelle(s) est (sont) la (les) proposition(s) exacte(s) ? A il s'agit d'une coupe coronale T2 Monsieur M.P., 34 ans, vous consulte pour la prise B il s'agit d'un macrodénome hypophysaire en charge d'une gynécomastie bilatérale associée à C il s'agit d'une coupe sagittale T1 des troubles de la libido évoluant depuis près d'un D il existe une compression du chiasma optique an. Le patient n'a pas d'antécédent notable. Il est E il existe un effondrement du plancher sellaire père de deux garçons de 9 et 7 ans. Il est chauffeur routier. Question 5 Les données de l'examen clinique sont les Quels sont, parmi ces résultats de champs visuels, suivantes : ceux compatibles avec l'IRM du patient ? • 90 kg, 178 cm ; • PA : 150/90 mm Hg ; FC : 78/min ; • absence d'adénopathie axillaire ; • gynécomastie bilatérale simple ; • présence de multiples molluscum pendulum au niveau de la base du cou ; • petit goitre hétérogène avec un renflement nodu- laire à droite.

Cas cliniques 472 A champ visuel 1 C le traitement par metformine est contre-indi- B champ visuel 2 qué du fait d'un possible syndrome d'apnées du C champ visuel 3 sommeil D champ visuel 4 E champ visuel 5 D ce diabète pourrait être corrigé par le traitement de son acromégalie Question 6 Le bilan préopératoire est le suivant : E les mesures hygiéno-diététiques sont les mêmes • glycémie à jeun : 1,5 g/l ; HbA1c : 7,3 % ; que pour un diabète de type 2 • triglycérides  : 2,5  g/l ; LDL-cholestérol  : 1,2  g/l ; Question 8 HDL-cholestérol : 0,32 g/l ; Quel(s) examen(s) paraclinique(s) prescrivez-vous dans • NFS  : leucocytes  : 8,88  G/l (N =  4,00-10,00) ; le cadre de son bilan d'acromégalie ? A oxymétrie nocturne hématies  : 4,74  T/l (N =  4,50-6,00) ; hémoglo- B coloscopie bine : 136 g/l (N = 130-170) ; hématocrite : 42 % ; C gastroscopie VGM : 89,7 fl (N = 80-100) ; plaquettes : 268 G/l D échographie thyroïdienne (N = 150-400). E échocardiographie Comment interprétez-vous les résultats ? A il présente un diabète de type 2 Question 9 B il présente une insulinorésistance due à l'excès de GH Quel(s) examen(s) prescrivez-vous pour explorer son C il présente un risque cardiovasculaire augmenté goitre ? D il présente une anémie macrocytaire A TSHus seule E le syndrome métabolique est indépendant de B TSHus et T4L l'acromégalie C scintigraphie thyroïdienne D échographie thyroïdienne Question 7 E iodurie Concernant son diabète, quelle(s) est (sont) la (les) proposition(s) exacte(s) ? Question 10 A une normalisation de l'HbA1c est indispensable Devant le bilan thyroïdien suivant : TSHus < 0,01 mUI/l et T4L = 26 pmol/l (N = 12-22), vous avez prescrit une avant toute prise en charge chirurgicale de son scintigraphie thyroïdienne dont voici le résultat. adénome hypophysaire B le traitement de son diabète impose une insulinothérapie

Cas cliniques 25 Quel(s) est (sont) le(s) diagnostic(s) compatible(s) avec • FSH : 4 UI/l (N = 2-7) ; 473 cette scintigraphie ? • LH : 2,5 UI/l (N = 2-6) ; A une maladie de Basedow • testostérone totale : 4 nmol/l (N = 10,4-26). Entraînement B un goitre multihétéronodulaire toxique Quelle prise en charge de son désir de paternité lui C un adénome toxique proposez-vous ? D un adénome hypophysaire avec co-sécrétion de A traitement androgénique par testostérone injectable B traitement par gonadotrophines injectables GH et de TSH C fécondation in vitro avec sperme de donneur E une thyroïdite subaiguë D aucun traitement, la normalité des gonadotro- Question 11 phines montre que l'axe gonadotrope est en cours Concernant cette hyperthyroïdie, quelle(s) est (sont) de normalisation l'(les) option(s) thérapeutique(s) ? E évaluation du spermogramme afin de juger du A thyroïdectomie totale degré d'altération de la spermatogenèse B antithyroïdiens de synthèse pendant 12 à 18 mois C abstention thérapeutique car elle régressera après Question 14 Finalement, quelle(s) étai(en)t l'(les) étiologie(s) traitement de son acromégalie possible(s) de sa gynécomastie initiale ? D traitement par iode radioactif A une hyperprolactinémie E lobo-isthmectomie droite B une hypothyroïdie centrale C un hypogonadisme hypogonadotrope Question 12 D une hyperthyroïdie Monsieur M.P. a été opéré de son adénome hypophy- E un déficit corticotrope saire il y a un mois. Quel(s) test(s) réalisez-vous pour vérifier l'absence de Question 15 déficit postopératoire ? Devant le jeune âge du patient et le volume initial de A test d'hypoglycémie insulinique avec dosage du la tumeur, vous évoquez une éventuelle prédisposi- tion génétique. cortisol plasmatique Quelle(s) est (sont), parmi les suivantes, la (les) B test à la TRH avec dosage de la TSH pathologie(s) associée(s) à l'acromégalie dans le cadre C test à la GnRH (LHRH) avec dosage de la LH et de d'une mutation du gène NEM1 ? A carcinome médullaire thyroïdien la FSH B hypothyroïdie d'Hashimoto D test d'hyperglycémie provoquée orale avec le C insulinome D maladie d'Addison dosage de la GH E hyperparathyroïdie primaire E test à la dexaméthasone avec dosage du cortisol Cas clinique 25 Question 13 Trois mois après l'opération, alors que l'acromégalie Un homme de 23 ans consulte pour une hypertrophie est considérée comme guérie, Monsieur M.P. se plaint sensible du sein gauche. Il a noté son apparition vers toujours de troubles de la libido et vous fait part de 14 ans mais son pédiatre avait annoncé que les choses son désir de paternité. Son bilan gonadotrope est le rentreraient spontanément dans l'ordre. Ses parents suivant : sont étonnés du fait qu'il ait continué à grandir tardi- vement. Il ne se plaint d'aucun symptôme. L'examen clinique fait par le médecin généraliste qui l'adresse à l'endocrinologue a retrouvé  : taille de 183  cm, poids 78 kg, pression artérielle 135/85 mm Hg, pouls 76/min. Question 1 Quelle(s) est (sont) la (les) proposition(s) exacte(s) ? A une gynécomastie vraie correspond à une prolifé- ration de tissu glandulaire ou adipeux dans le terri- toire mammaire B une gynécomastie vraie correspond à une prolifé- ration de tissu exclusivement glandulaire dans le territoire mammaire C une gynécomastie de survenue pubertaire justifie un bilan hormonal systématique dès sa découverte D une gynécomastie liée à une hyperœstrogénie locale est toujours bilatérale E une gynécomastie vraie peut être douloureuse

Cas cliniques Question 2 Vous évoquez : Concernant le bilan d'une gynécomastie, quelle(s) est A un hypogonadisme central (sont) la (les) proposition(s) exacte(s) ? B un hypogonadisme périphérique A une tuméfaction dure et excentrée par rapport au C une hyperthyroïdie mamelon doit faire évoquer un cancer du sein D une tumeur germinale testiculaire B une hypertrophie mammaire bilatérale doit faire E une insensibilité aux androgènes évoquer le diagnostic différentiel d'adipomastie Question 7 C une tuméfaction centrée sur l'aréole et mobile Dans ce contexte, il doit réaliser un examen à visée sur les plans profonds évoque la présence de tissu étiologique : lequel ? glandulaire bénin A une échographie des testicules D la palpation est suffisante dans tous les cas pour B un dosage de la TeBG, protéine de transport des affirmer une gynécomastie stéroïdes sexuels E une échographie ou une mammographie est géné- C un dosage de prolactine D une IRM hypophysaire ralement nécessaire pour affirmer une gynécomastie Question 3 E un caryotype L'échographie a confirmé la présence de tissu glan- Question 8 dulaire mammaire gauche. Le patient est interrogé À ce stade, quel(s) traitement(s) doi(ven)t être par l'endocrinologue  : celui-ci recherche une prise proposé(s) ? A un anti-œstrogène de type tamoxifène médicamenteuse pouvant expliquer la gynécomastie. B un inhibiteur de l'aromatase C la testostérone par voie intramusculaire Laquelle (lesquelles) peut (peuvent) être potentielle- D la dihydrotestostérone par voie percutanée E la chirurgie plastique mammaire ment impliquée(s) dans son apparition ? A une prise d'œstrogènes B une prise d'androgènes anabolisants C une prise de marijuana ou d'héroïne D une prise d'antirétroviraux Question 9 E une prise de corticoïdes Le traitement choisi va permettre : Question 4 A le développement de la pilosité 474 Le patient ne prend aucun de ces composés B le développement de la musculature médicamenteux. C le développement du volume testiculaire Concernant les mécanismes possibles à l'origine d'une D de protéger l'os de l'ostéoporose gynécomastie, quelle(s) est (sont) la (les) proposition(s) E de réduire la gynécomastie exacte(s) ? Question 10 A une hypothyroïdie peut être en cause B une hyperthyroïdie peut être en cause Le patient s'interroge sur sa fertilité. Un spermogramme C une insuffisance androgénique peut être en cause D une insensibilité cellulaire aux androgènes peut a été réalisé qui retrouve 1 million/mm3 de spermato- être en cause zoïdes (N : 40-200 millions/mm3). En cas de demande E une hypersécrétion testiculaire ou surrénalienne de procréation du couple, on pourra leur proposer : d'œstrogènes peut être en cause A d'augmenter la dose mensuelle de testostérone B un traitement par les gonadotrophines hCG/FSH Question 5 C une fécondation in vitro classique D une fécondation par micro-injection intra-ovocytaire Le patient est examiné par l'endocrinologue qui note de spermatozoïdes testiculaires extraits à partir une pilosité clairsemée au visage et au niveau pubien. d'une biopsie testiculaire (TESE-ICSI) Sa verge est développée, ses testicules petits (4  ml E une adoption droit et gauche). Ces signes peuvent faire évoquer : Cas clinique 26 A une tumeur testiculaire à cellules de Leydig sécrétantes B une insuffisance testiculaire primaire Un patient âgé de 75 ans a un traitement par amio- C une tumeur hypophysaire de type prolactinome darone de ses accès de tachycardie ventriculaire. Il a D une tumeur hypothalamique de type craniopha- un défibrillateur implanté. Il avait, avant le traitement, une TSH normale. Il est diabétique avec un traitement ryngiome de gliclazide LM 30, 2 cp. par jour. E une tumeur surrénalienne sécrétante Question 6 Question 1 L'endocrinologue réalise des dosages sanguins qui retrouvent dans le plasma : Pour surveiller la fonction thyroïdienne de ce patient, • testostérone : 2,5 ng/ml (N = 3-9) ; • œstradiol : 10 pg/ml (N = 10-40) ; vous dosez régulièrement : • LH : 15 mU/ml (N = 1-7) ; A TSH • β-hCG indétectable (N < 0,1) ; B TSH, T4L • TSH : 3 mU/l (N = 0,1-4). C TSH, T4L, T3L D anticorps anti-récepteurs de la TSH E thyroglobuline

Cas cliniques 25 Question 2 Vous évoquez : 475 Son médecin traitant demande régulièrement le A une hyperthyroïdie sur nodule toxique aggravée dosage de TSH, T4L, T3L. Entraînement Chez ce patient euthyroïdien, sous amiodarone au par la surcharge iodée long cours, vous vous attendez à trouver : B une maladie de Basedow aggravée par la sur- A une TSH basse B une TSH élevée charge iodée C une TSH normale avec une T4L haute et une T3L haute C une hyperthyroïdie due à l'amiodarone de type I : D une TSH normale avec une T4L haute et une T3L basse E une TSH normale avec une T4L basse et une T3L haute fonctionnelle D une hyperthyroïdie due à l'amiodarone de type II : Question 3 Le patient consulte en urgence car il fait des malaises lésionnelle et son défibrillateur a déclenché plusieurs chocs E une thyroïdite subaiguë de De Quervain électriques. Quelle(s) est (sont) la (les) proposition(s) exacte(s) ? Question 7 A vous devez arrêter l'amiodarone En faveur de l'hyperthyroïdie de type II, vous retenez : B vous devez doser la TSH A l'âge du patient C vous devez doser l'iodurie des 24 heures B l'absence d'antécédent thyroïdien D vous devez demander une scintigraphie thyroïdienne C la présence d'un nodule E vous devez diminuer la dose hebdomadaire D la scintigraphie blanche E le délai d'apparition de l'hyperthyroïdie après l'in- d'amiodarone troduction de l'amiodarone Question 4 Le diagnostic d'hyperthyroïdie est porté. Question 8 Vous observez : Vous avez retenu deux hypothèses : la thyroïdite subai- A une TSH freinée guë et l'hyperthyroïdie due à l'amiodarone de type II. B une T4L élevée, une T3L élevée Pour les distinguer, vous demandez chez ce patient : C une T4 élevée, une T3L basse A le dosage de la CRP D une T4L normale, une T3L élevée B le dosage de la thyroglobuline E une thyroglobuline effondrée C le dosage de l'iodurie des 24 heures D le dosage des anticorps anti-thyroperoxydase Question 5 E le dosage des anticorps anti-récepteurs de la TSH Pour préciser le mécanisme de l'hyperthyroïdie, vous demandez : Question 9 A une échographie thyroïdienne C'est une hyperthyroïdie due à l'amiodarone de B une scintigraphie thyroïdienne à l'iode 123 type II. La T4L est à 84 pmol/l (N < 20) ; la T3L est à C une iodurie des 24 heures 18 pmol/l (N < 6,5). D des anticorps anti-récepteurs de la TSH Vous décidez : E un dosage de calcitonine A arrêt impératif de l'amiodarone et surveillance B arrêt impératif de l'amiodarone et introduction de Question 6 L'échographie montre une thyroïde de taille normale, propylthio-uracyl très hypoéchogène, apparaissant non vasculaire au C arrêt de l'amiodarone et introduction de corticoïdes doppler. Il existe un nodule. La scintigraphie à l'iode D arrêt de l'amiodarone et traitement par iode 131 123 donne cette image. E après accord du cardiologue, arrêt de l'amiodarone et introduction de corticoïdes Question 10 Après 10  jours de corticoïdes, la T4L a diminué à 56  pmol/l (N < 20), la T3L à 12  pmol/l (N < 5,6). La glycémie est montée à 3,5 g/l. Quelle(s) est (sont) la (les) proposition(s) exacte(s) ? A vous interrompez les corticoïdes B vous maintenez la corticothérapie et ajoutez de la metformine C vous maintenez la corticothérapie et augmentez le gliclazide à 3 cp. par jour D vous maintenez la corticothérapie, vous arrêtez le gliclazide et introduisez un analogue du GLP-1 E vous maintenez la corticothérapie et remplacez le gliclazide par une insulinothérapie Question 11 Le traitement par corticoïdes a amené le retour à une fonction thyroïdienne normale. Des accès de tachy- cardie ventriculaire continuent à survenir (déclen- chement du défibrillateur). Le cardiologue souhaite reprendre l'amiodarone. L'iodurie est normalisée.

Cas cliniques Quelle(s) est (sont) la (les) proposition(s) exacte(s) ? anormalement élevés, quelle(s) est (sont) votre (vos) A vous autorisez la reprise de l'amiodarone principale(s) hypothèse(s) diagnostique(s) ? B vous contre-indiquez définitivement l'amiodarone A insulinome C vous autorisez la reprise de l'amiodarone à B prise cachée de sulfamides hypoglycémiants C tumeur fibreuse solitaire du foie sécrétant de demi-dose D vous discutez un traitement préventif par iode 131 l'IGF-2 D injection cachée d'insuline avant reprise de l'amiodarone E aucun des diagnostics ci-dessus E vous discutez une thyroïdectomie préventive avant Question 5 reprise de l'amiodarone Si vous obtenez, lors du jeûne, une glycémie à 0,38  g/l associée à une insulinémie et un peptide  C Cas clinique 27 bas, quelle(s) est (sont) votre (vos) principale(s) hypothèse(s) diagnostique(s) ? Monsieur F., 54 ans, consulte pour des malaises qu'il A insulinome qualifie d'hypoglycémiques associant des sueurs, une B prise cachée de sulfamides hypoglycémiants asthénie intense, des palpitations. Ces malaises sur- C tumeur fibreuse solitaire du foie sécrétant de viennent essentiellement le dimanche matin lorsqu'il s'octroie une grasse matinée. Il décrit également une l'IGF-2 perte de connaissance survenue récemment à la fin D injection cachée d'insuline d'un match de tennis. E aucun des diagnostics ci-dessus Question 1 Question 6 Quel(s) est (sont) le(s) signe(s) à rechercher à l'interro- Si vous obtenez, lors du jeûne, une glycémie à gatoire évocateur(s) d'une hypoglycémie organique ? 0,65  g/l associée à une insulinémie et un peptide  C A symptomatologie disparaissant rapidement après élevés, quelle(s) est (sont) votre (vos) principale(s) l'ingestion de sucres rapides hypothèse(s) diagnostique(s) ? B cauchemars nocturnes fréquents A insulinome C perte de connaissance brutale d'allure syncopale, B prise cachée de sulfamides hypoglycémiants sans prodromes C tumeur fibreuse solitaire du foie sécrétant de 476 D malaise survenant essentiellement 2  heures après l'IGF-2 un repas riche en glucides simples D injection cachée d'insuline E perte d'urines au décours du malaise E aucun des diagnostics ci-dessus Question 2 Question 7 Quel(s) est (sont) l'(les) antécédent(s) qui pourrai(en)t L'épreuve de jeûne est en faveur d'un insulinome. vous orienter dans votre diagnostic étiologique des Quel(s) examen(s) peut (peuvent) être utile(s) dans le hypoglycémies ? diagnostic topographique de l'insulinome ? A alcoolisme chronique compliqué d'une dénutrition A scintigraphie au MIBI B scintigraphie à la MIBG sévère C scintigraphie à l'Octréoscan® B polykystose rénale compliquée d'une insuffisance D scanner pancréatique E IRM pancréatique rénale préterminale F échoendoscopie pancréatique C nodule thyroïdien toxique traité par lobo-­isthmectomie Question 8 gauche il y a 1 an Concernant l'insulinome, quelle(s) est (sont) la (les) D maladie de Cushing traité par adénomectomie proposition(s) exacte(s) ? A il s'agit le plus souvent d'une tumeur de petite hypophysaire il y a 3 mois E hyperparathyroïdie primaire opérée il y a 20 ans taille B il n'est jamais métastatique Question 3 C il peut être multiple Vous décidez de réaliser un bilan complémentaire. D il ne peut pas être présent chez un enfant Quel(s) est (sont) l'(les) examen(s) que vous propose- riez à Monsieur F. ? Question 9 A hyperglycémie provoquée par voie orale Le scanner pancréatique met en évidence une lésion B épreuve de jeûne de 48 heures de la queue du pancréas évocatrice d'une tumeur C épreuve de jeûne de 72 heures neuroendocrine mesurant 2  cm. Vous décidez de lui D glycémie veineuse, insulinémie, peptide C plasma- proposer une chirurgie pancréatique par énucléation. En reprenant l'interrogatoire, Monsieur F. vous signale tique lors d'un malaise que sa sœur de 2 ans plus âgée a été opérée à l'âge de E glycémie capillaire, insulinémie et peptide C plas- 40 ans d'un macroadénome hypophysaire non sécré- tant compliqué d'une altération du champ visuel. matique lors d'un malaise Question 4 Si vous obtenez, lors du jeûne, une glycémie à 0,42  g/l associée à une insulinémie et un peptide  C

Cas cliniques 25 Quelle pathologie devez-vous évoquer dans cette Question 3 477 famille ? Voici les résultats de son bilan : A maladie de von Hippel-Lindau • β-hCG : négatives ; B néoplasie endocrinienne multiple de type 1 • œstradiol : < 20 pg/ml ; C néoplasie endocrinienne multiple de type 2 • FSH : 2 UI/l (N = 1-8) ; D sclérose tubéreuse de Bourneville • LH : 3 UI/l (N = 2-7) ; E neurofibromatose de type 1 • prolactine : 35 ng/ml (N < 20). Concernant ce bilan, quelle(s) est (sont) la (les) Question 10 proposition(s) exacte(s) ? Quel dosage biologique simple peut permettre de A il existe un hypogonadisme hypogonadotrophique conforter votre hypothèse diagnostique ? B le bilan évoque une insuffisance ovarienne primi- A glycémie veineuse à jeun B kaliémie tive C créatinémie C l'hyperprolactinémie peut être d'origine médica- D calcémie E magnésémie menteuse F chlorémie D l'hypogonadisme est secondaire à l'hyperprolacti- Question 11 némie Votre hypothèse diagnostique génétique est confirmée. E un macroadénome à prolactine est suspecté Quelle(s) autre(s) lésion(s) duodénopancréatique(s) peut (peuvent) être associée(s) à l'insulinome ? Question 4 A glucagonome Lors de cette même consultation, elle décrit un syn- B VIPome drome polyuro-polydipsique apparu depuis quelques C gastrinome semaines. Elle boit environ 3 litres par jour et se lève D somatostatinome la nuit pour boire également 2 à 3  litres, avec une E toutes les lésions précédentes diurèse identique. Vous avez éliminé un diabète sucré et vous évoquez Cas clinique 28 un syndrome polyuro-polydipsique. Quel(s) est (sont) Entraînement l'(les) élément(s) compatible(s) avec un diagnostic de Une patiente de 35 ans consulte pour une aménorrhée. diabète insipide ? Elle n'a pas d'antécédent particulier, elle est G2P2, sans A natrémie à 125 mmol/l difficulté à concevoir, ni diabète gestationnel. B natrémie à 135 mmol/l Son traitement comporte uniquement de l'Efferalgan C natrémie à 148 mmol/l codéiné® pour des arthralgies, avec parfois du dom- D osmolarité à 295 mOsm/l péridone. Elle se dit plus fatiguée depuis quelques E osmolarité urinaire à 300 mOsm/l mois et parfois essoufflée à l'effort. Elle a eu ses pre- mières règles vers 15  ans, avec des cycles toujours Question 5 réguliers par la suite. Depuis 6 mois, elle n'a pas eu de Pour affirmer ce diagnostic, elle est hospitalisée pour règles. À l'examen, elle pèse 56 kg pour 1,65 m. Sa un test de restriction hydrique, dont voici le résultat : PA est à 126/67 mm Hg. Elle n'a pas d'hirsutisme, pas de vergetures, ni prise de poids. Elle ne décrit aucune T 0 T 1 h T 4 h Après bouffée de chaleur. Elle présente une galactorrhée injection minime provoquée unilatérale. Osmolarité 295 293 de DDAVP plasmatique Question 1 (mOsm/l) 142 148 300 290 À ce stade, concernant l'aménorrhée, quelle(s) est 300 400 (sont) la (les) étiologie(s) à évoquer ? Natrémie 200 180 150 143 A syndrome des ovaires polykystiques (mmol/l) 300 70 B insuffisance gonadotrope 212 478 C prolactinome Diurèse D ménopause précoce horaire (ml) E syndrome de Turner Osmolarité urinaire (mOsm/l) Question 2 lequel (lesquels) Quelle(s) est (sont) la (les) proposition(s) exacte(s) ? A il existe un déficit partiel en ADH Parmi les dosages suivants, B le diagnostic de diabète insipide est incertain C vous suspectez un diabète insipide néphrogénique prescrivez-vous ? D la réponse à l'injection d'ADH élimine un diabète A œstradiol B FSH et LH insipide complet C progestérone E il s'agit d'un diabète insipide central D 17-OH-progestérone E prolactine

Cas cliniques Question 6 nodules pulmonaires et adénopathies médiastinales Devant cette association aménorrhée et dia- avec une formule lymphocytaire au LBA. Compte bète insipide central, vous prescrivez une IRM tenu de l'atteinte pulmonaire, une corticothérapie est hypothalamo-hypophysaire. instaurée à 80 mg de prednisone par jour. La patiente A priori, quelle(s) pathologie(s) peut (peuvent) expli- revient vous voir en consultation d'endocrinologie quer ce tableau clinique ? pour évaluation des fonctions hypophysaires. A un microadénome hypophysaire Concernant la supplémentation gonadique, votre B un macroadénome hypophysaire invasif patiente étant demandeuse d'un traitement hormonal C un craniopharyngiome substitutif, quelle(s) est (sont) la (les) thérapeutique(s) D une infiltration granulomateuse de la région que vous allez lui proposer ? A un œstrogène oral de J1 à J25 hypothalamo-hypophysaire B un œstrogène percutané E une tumeur germinale de la région hypothalamo- C un œstrogène en patch hypophysaire Question 7 Voici son IRM hypophysaire. 1 2 3 4 478 5 Indiquez les structures anatomiques correctement D un progestatif est nécessaire pour éviter le risque identifiées : de cancer du sein A « 1 » est le troisième ventricule B « 2 » correspond à l'un des nerfs optiques E il est inutile de lui prescrire un progestatif car elle a C « 3 » indique l'hypophyse plus de 35 ans D « 4 » correspond à la veine jugulaire E « 5 » est le sinus sphénoïdal pneumatisé Question 11 Quelle(s) réponse(s) lui donnez-vous si elle souhaite Question 8 une nouvelle grossesse ? Devant cette image, laquelle (lesquelles) des patholo- A elle doit être adressée dans un service de procréa- gies suivantes retenez-vous ? A un macroadénome hypophysaire invasif tion médicale assistée B un craniopharyngiome B un traitement par citrate de clomiphène sera pro- C une histiocytose D une sarcoïdose posé en première intention E un myélome C des injections de gonadotrophines sont une option Question 9 thérapeutique Pour avancer dans votre démarche diagnostique, D une pompe à GnRH est une option thérapeutique quel(s) examen(s) proposez-vous en première inten- E elle sera inscrite sur la liste pour un don tion dans l'hypothèse d'une sarcoïdose ? A une biopsie hypophysaire d'ovocytes B un scanner thoracique C un PET-scan Question 12 D un dosage d'ACE Concernant le traitement de son diabète insipide par E une fibroscopie bronchique avec lavage bronchiolo- vasopressine, quelle(s) est (sont) la (les) proposition(s) exacte(s) ? alvéolaire A il faut augmenter la dose si le syndrome polyuro- Question 10 polydipsique persiste Le diagnostic de sarcoïdose hypothalamo-hypophysaire B un objectif du traitement est une natrémie normale est confirmé devant une élévation de l'ACE, des micro- C un objectif du traitement est une natriurèse supé- rieure à 170 mmol par 24 heures D une hyponatrémie est le signe d'un surdosage en vasopressine E le traitement pourra être interrompu après 1 mois

Cas cliniques 25 Question 13 D correspondent à un cancer thyroïdien dans 10 à 479 Après deux ans, compte tenu d'une prise de poids 15 % des cas majeure (85  kg) avec apparition d'une hypertension Entraînement artérielle, le pneumologue envisage un changement E sont le plus souvent bénins de traitement. Il a diminué à 10 mg la prednisone et introduit du méthotrexate. Question 2 Pour évaluer la fonction corticotrope dans ce contexte, L'exploration d'un nodule thyroïdien commence par : quel(s) examen(s) demandez-vous ? A une scintigraphie thyroïdienne A un dosage de cortisol à 8 h B un dosage de TSH B un test au Synacthène® 250 mg C un dosage de thyroglobuline C une hypoglycémie insulinique D un dosage de calcitonine D un test à la Métopirone® E un scanner cervicothoracique E un test au CRH Question 3 Question 14 La TSH est normale à 1,06 mUI/l. L'échographie cer- Cinq ans plus tard, la maladie est bien contrôlée sous vicale retrouve un nodule thyroïdien, unique, isthmo- méthotrexate et l'IRM hypothalamo-hypophysaire ne lobaire gauche de 12 mm de diamètre, hypoéchogène, retrouve plus de lésions. hétérogène, comportant des microcalcifications, au Concernant l'atteinte hypothalamo-hypophysaire, sein d'une thyroïde modérément hyperplasiée. Il n'y quelle(s) est (sont) la (les) proposition(s) exacte(s) ? a pas de ganglion suspect. A il existe une corrélation entre les images et les Vous proposez une cytoponction échoguidée du nodule : A en raison de la taille du nodule déficits B en raison du caractère unique du nodule B une récupération hypophysaire est possible C en raison des caractéristiques échographiques du C la fonction gonadotrope est d'une façon générale nodule la fonction la plus fragile D en raison des antécédents de radiothérapie cervicale D on ne peut pas juger de la guérison du diabète E en raison d'un antécédent de cancer insipide sans arrêter la supplémentation Question 4 E l'hyperprolactinémie de déconnexion est défini- La cytoponction retrouve un nodule suspect de can- cer, classé en catégorie 5 de Bethesda. Une interven- tive tion chirurgicale est proposée. Avant la chirurgie, on complète les examens par : Question 15 A un dosage de thyroglobuline Le bilan hormonal réalisé est le suivant : B un dosage de calcitonine • œstradiol  : 10 pg/ml (N = 50-200 en phase C un scanner cervicothoracique à la recherche de folliculaire) ; métastases • LH : 1 UI/l (N = 7-56) ; D une biopsie chirurgicale du nodule • FSH : 2 UI/l (N = 2-8) ; E aucun examen n'est nécessaire • PRL : 7 ng/ml (N = 12-22) ; • TSH : 1,2 UI/l ; Question 5 • T4L : 8 pmol/l (N = 12-22) ; Une thyroïdectomie totale associée à un curage gan- • ACTH : 2 pg/ml (N = 7-70) ; glionnaire central est pratiquée. Un traitement de • cortisol : 120 nmol/l (N = 200-600) ; L-T4 est mis en place. • GH : 0,1 ng/ml (N = 0,01-3,6) ; Le lendemain de l'intervention, le patient se plaint de • IGF-1 : 86 ng/ml (N = 100-350). crampes douloureuses. Vous demandez en priorité : Quelle est votre conclusion ? A un dosage de potassium B un dosage de magnésium Cas clinique 29 C un dosage de calcémie D un dosage de TSH Anthony, âgé de 27  ans, consulte pour un nodule E un électromyogramme thyroïdien qu'il a découvert en faisant sa toilette. Sa mère est traitée pour une insuffisance thyroïdienne. Il Question 6 a été traité dans l'enfance pour une maladie de Hodg- Le bilan retrouve une calcémie à 1,75  mmol/l (N = kin dont il est guéri. 2,20-2,60), suggérant une hypoparathyroïdie La palpation cervicale retrouve un nodule ferme d'en- postopératoire. viron 1 cm du lobe gauche sur thyroïde palpable. Il n'y Quelle(s) est (sont) la (les) proposition(s) exacte(s) ? a pas d'adénopathie. A il s'agit d'une complication possible de toute Question 1 chirurgie thyroïdienne, qu'il s'agisse d'une thyroï- Les nodules thyroïdiens : dectomie partielle ou totale A ont une prévalence qui augmente avec l'âge B ce risque doit avoir été expliqué au patient avant B sont plus fréquents chez les hommes l'intervention C sont souvent découverts, aujourd'hui, de façon C cette complication n'est observée que lorsque l'équipe chirurgicale est peu entraînée fortuite lors d'examens d'imagerie

Cas cliniques D elle est plus fréquente en cas de réintervention ou 1 μg/l sous stimulation. Anthony se plaint de douleurs de curage ganglionnaire cervicales intenses 48 heures après la dose. E elle est toujours définitive Quelle(s) est (sont) la (les) proposition(s) exacte(s) ? F elle peut justifier un traitement vitaminocalcique A la fixation correspond vraisemblablement à une au long cours métastase cervicale fixante Question 7 B la fixation correspond vraisemblablement à un reli- L'examen anatomopathologique conclut à un carci- nome papillaire de 11 mm de diamètre, classé pT1b. quat thyroïdien banal Il existe un microenvahissement de deux des huit gan- C il faut craindre une angine secondaire à une aplasie glions retirés lors du curage (N1a). A vous envoyez un courrier au patient pour l'infor- induite par l'iode radioactif D il s'agit d'une réaction inflammatoire thyroïdienne mer que le diagnostic est confirmé induite par l'iode radioactif E un traitement corticoïde transitoire peut être proposé B vous revoyez le patient en consultation pour lui Question 11 annoncer le diagnostic Un an après la dose, la thyroglobuline sous stimulation C le dossier doit être présenté en RCP est indétectable, l'échographie cervicale est normale. D ce cancer thyroïdien est de bon pronostic, il n'est Quelle(s) est (sont) la (les) proposition(s) exacte(s) ? pas nécessaire de présenter le dossier en RCP A le patient est en rémission de son cancer thyroïdien E ce cancer thyroïdien est de bon pronostic, il n'est B aucune surveillance ultérieure n'est nécessaire pas nécessaire de l'inquiéter inutilement en lui C il faut doser la TSH tous les ans pour s'assurer que communiquant le diagnostic le traitement de L-T4 est bien adapté Question 8 D il n'est plus nécessaire de doser la thyroglobuline Les résultats de l'examen anatomopathologique vous E il faut continuer à surveiller la thyroglobuline tous inspirent les réflexions suivantes : A il s'agit d'une forme de bon pronostic compte tenu les ans F un dosage des anticorps anti-thyroglobuline est de l'âge systématiquement couplé au dosage de Tg B il s'agit d'une forme de bon pronostic compte tenu Question 12 de l'histologie papillaire 480 C l'envahissement ganglionnaire est fréquent dans Le contrôle de TSH sous L-T4 100  μg par jour est à 8 mUI/l pour un objectif de TSH entre 0,3 et 2 mUI/l. les cancers papillaires et chez les sujets jeunes Quelle(s) est (sont) la (les) proposition(s) exacte(s) ? D l'existence d'un envahissement ganglionnaire A il faut augmenter le traitement de L-T4 à 125 μg majore le risque de récidive par jour E l'existence d'un envahissement ganglionnaire B il faut diminuer le traitement de L-T4 à 75 μg par jour majore le risque de mortalité C un ECG et un avis cardiologique sont nécessaires Question 9 avant modification du traitement Un traitement adjuvant par l'iode radioactif (IRAthé- D le traitement peut être modifié sans précaution rapie) est décidé. Il consiste en l'administration d'une activité de 1 110 MBq (30 mCi) sous TSH recombinante. particulière chez ce sujet jeune Quelle(s) est (sont) la (les) proposition(s) exacte(s) ? E le contrôle de TSH doit être réalisé 8 jours après la modification de dose A une hospitalisation en chambre radioprotégée est Question 13 Si la scintigraphie post-dose avait montré des métas- nécessaire tases pulmonaires : B le traitement peut s'effectuer en ambulatoire en A le risque de mortalité aurait été très élevé B le traitement de L-T4 devrait être maintenu à dose prenant des précautions vis-à-vis de l'entourage frénatrice de la TSH familial et professionnel C une nouvelle dose d'iode radioactif aurait été indiquée C une stimulation de la TSH (arrêt de L-T4 ou adminis- D une chimiothérapie aurait été indiquée E une radiothérapie externe aurait été indiquée tration de TSH recombinante) est nécessaire pour Question 14 que les résidus thyroïdiens fixent l'iode radioactif Au cours du suivi, 3  ans plus tard, l'échographie D une préparation à base de soluté riche en iode est découvre un ganglion jugulo-carotidien gauche sus- pect de 12 mm de diamètre. proposée pour limiter l'irradiation Quelle(s) est (sont) la (les) proposition(s) exacte(s) ? E une scintigraphie corporelle totale est réalisée A on propose une biopsie chirurgicale B on propose une ablation chirurgicale quelques jours après la dose pour objectiver les C on propose une ponction du ganglion pour ana- résidus thyroïdiens F un dosage de calcitonine est systématiquement réalisé sous stimulation de la TSH G un dosage de thyroglobuline est systématique- ment réalisé sous stimulation de la TSH Question 10 lyse cytologique couplée à un dosage de Tg in situ La scintigraphie post-dose retrouve une fixation cer- D on propose une nouvelle dose d'iode radioactif vicale isolée, médiane. La thyroglobuline est basse à E on demande un dosage de thyroglobuline

Cas cliniques 25 Question 15 • TSHus : 2,4 mUI/l (N = 0,4-4) ; 481 La ponction du ganglion a montré une simple hyper- • cholestérol total : 1,80 g/l ; triglycérides : 0,90 g/l ; plasie lymphoïde. Entraînement Anthony vit en couple et envisage d'avoir un enfant. LDL-cholestérol : 1,14 g/l. Quelle(s) est (sont) la (les) proposition(s) exacte(s) ? A vous lui dites d'attendre encore au moins 2  ans Question 1 Comment analysez-vous le résultat de la glycémie ? compte tenu des risques de récidive du cancer A Monsieur V. est diabétique thyroïdien B il faut refaire un dosage de glycémie à jeun pour B vous lui donnez le feu vert C vous le prévenez de risques malformatifs potentiels confirmer le diagnostic de diabète du fait de la dose d'iode radioactif C il faut faire un dosage d'hémoglobine glyquée D vous le prévenez des risques de stérilité du fait de la dose d'iode radioactif pour confirmer le diagnostic de diabète E vous le prévenez du caractère potentiellement héré- D l'anémie fausse le résultat de la glycémie ditaire de son cancer papillaire qui imposera une E le même résultat glycémique à n'importe quel surveillance thyroïdienne rapprochée chez l'enfant moment de la journée permettrait d'affirmer le Cas clinique 30 diagnostic de diabète Monsieur V., 61 ans, vient vous voir dans le cadre de Question 2 perturbations du bilan biologique : Quel(s) examen(s) réalisez-vous en urgence ? • antécédents familiaux : inconnus pour la plupart, A HbA1c B bandelette urinaire issu d'une fratrie de quatre enfants, son frère était C fond d'œil diabétique et serait décédé d'un cancer du foie. Il a D ECG une fille de 32 ans qui a eu un diabète gestationnel E échographie abdominale lors de sa deuxième grossesse l'an dernier ; • antécédents médicaux : BPCO post-tabagique ; Question 3 • antécédents chirurgicaux  : prothèse de l'épaule La bandelette urinaire retrouve : glycosurie « +++ » ; droite (accident de travail), chirurgie de varices. protéines  : « traces » ; cétonurie « ++ » ; leucocytes  : Le tabagisme est sevré depuis 3  ans. Il travaillait « 0 » ; nitrites : « 0 » ; sang : « 0 ». L'ECG ne retrouve jusqu'à il y a 2 ans dans un restaurant (cuisinier). Il est pas de signe d'hypokaliémie. La glycémie capillaire est breton d'origine. Il a peu d'activité physique, passe à 3,54 g/l. la majorité de son temps devant son ordinateur. Il Monsieur  V. est adressé aux urgences ; des gaz du ne prend pas de traitement, sauf des β2-mimétiques sang sont réalisés, ne retrouvant pas d'acidose. Un inhalés si besoin. protocole « acidocétose » (réhydratation, insuline Depuis quelques mois, il a noté une asthénie présente IVSE, potassium IVSE) est mis en place. Au bout de dès le matin, s'aggravant au cours de la journée. Le 4 heures, l'ionogramme est normalisé et il n'y a plus poids est à 68 kg, la taille de 1,73 m. Il aurait perdu d'acétone. 4 kg au cours des derniers mois. Il vous rapporte aussi Monsieur  V. est hospitalisé dans le service d'endo- une augmentation de sa consommation de sodas crinologie pour la suite de la prise en charge. Le depuis environ 1  mois et plusieurs levers nocturnes bilan biologique réalisé le lendemain retrouve de pour uriner et boire un verre d'eau. Il n'a pas de nouveau une cytolyse > 3  N prédominant en ALAT ; plainte fonctionnelle par ailleurs. HbA1c : 11,2 % ; TP : 58 % ; CRP < 2 mg/l ; calcémie : À l'examen, vous constatez une hépatomégalie et des 2,24 mmol/l ; albumine : 34 g/l. hématomes au niveau des bras et des jambes (surve- Quels sont les deux examens qui vous paraissent nus sans traumatisme). les plus pertinents pour avancer dans le dia- Le bilan biologique est le suivant (réalisé à 8 h à jeun) : gnostic étiologique de ce diabète nouvellement • Na  : 145  mmol/l (N =  132-146) ; K  : 3,4  mmol/l diagnostiqué ? (N = 3,5-5) ; A dosage des autoanticorps du diabète de type 1 • créatinine : 50 μmol/l ; B dosage du cortisol libre urinaire des 24 heures et • hémoglobine : 10,1 g/dl ; VGM : 102 fl ; • plaquettes  : 109  G/l ; leucocytes  : 8  G/l ; formule créatininurie normale ; C dosage de l'IGF-1 • ASAT : 105 UI/l (N < 34) ; ALAT : 176 UI/l (N = 10- D mesure de l'élastase fécale 49) ; γ-GT : 184 UI/l (N < 73) ; bilirubine totale : 45 E réalisation d'une TDM abdominale (N < 21) ; bilirubine directe : 19 (N < 3) ; • glycémie : 3,62 g/l ; Question 4 La TDM abdominale ne retrouve pas d'anomalie pancréatique mais le parenchyme hépatique paraît hétérogène. Il reconnaît avoir consommé de l'alcool en excès, surtout quand il avait encore une activité professionnelle, non sevré actuellement.

Cas cliniques Le tableau présenté par Monsieur V. est compatible D dosage de la GH après test de freinage par hyper- avec : glycémie provoquée par voie orale A diabète de type 1 B diabète de type 2 E dosage du coefficient de saturation de la transferrine C diabète secondaire à une pancréatite D diabète secondaire à une hémochromatose Question 7 E diabète secondaire à une cirrhose éthylique Parmi les arguments en faveur d'une hémochroma- tose, vous recherchez : Question 5 A une pigmentation endobuccale Quel traitement mettez-vous en place pour le diabète, B des douleurs rachidiennes en relais de la seringue électrique d'insuline ? C des douleurs articulaires des genoux et des mains A metformine seule D une parotidomégalie B metformine + sulfamide hypoglycémiant E des signes d'hyperthyroïdie C metformine + insuline D glinides seul Question 8 E insuline seule Le dosage du cortisol à 8 h est à 18 μg/dl (N = 6-20). Le coefficient de saturation de la transferrine (CS-Tf) Question 6 est à 70 %. Vous avez mis en place un traitement par insuline lente Quel(s) examen(s) complémentaire(s) demandez-vous 26 U le soir, analogue rapide de l'insuline environ 6 U à ce stade ? le matin, 8 U à midi, 6 U le soir. Il est très motivé par un A analyse génétique du gène HFE sevrage en alcool, qui est démarré en hospitalisation. B nouveau dosage du CS-Tf après équilibration du Pour la première fois depuis son hospitalisation, vous êtes néanmoins intrigué par la coloration de sa peau, diabète qui vous semble mélanoderme. Voici sa photographie. C dosage du récepteur soluble de la transferrine D dosage de la ferritine E test au Synacthène® sur le cortisol Question 9 Vous avez suspecté une hémochromatose et demandé une analyse génétique du gène HFE. Concernant cette analyse génétique, quelle(s) est 482 (sont) la (les) proposition(s) exacte(s) ? A il faudra deux prélèvements indépendants à 3 mois d'intervalle B vous adressez le résultat au médecin traitant qui informe le patient C vous faites signer un consentement écrit D elle est prise en charge par la Sécurité sociale E le résultat, s'il retrouve une mutation, fera l'objet d'une déclaration obligatoire aux autorités de santé Quel(s) examen(s) complémentaire(s) demandez-vous ? Question 10 A dosage du cortisol à minuit Le génotypage du gène HFE pour Monsieur V. retrouve B dosage de l'ACTH à minuit C dosage de l'IGF-1 une mutation homozygote C282Y. Concernant la génétique de l'hémochromatose, quelle(s) est (sont) la (les) proposition(s) exacte(s) ? A il s'agit d'une maladie à transmission autosomique récessive B il s'agit d'une maladie dont la pénétrance est variable d'un individu à l'autre C l'anomalie génétique la plus fréquemment rencon- trée est une mutation C282Y du gène HFE à l'état homozygote D si l'épouse de Monsieur V. est aussi porteuse hété- rozygote de la mutation, leurs enfants seront tous atteints d'hémochromatose E vous devez contacter directement la fille de Mon- sieur V. pour lui proposer le test génétique

Cas cliniques 25 Question 11 Comment interprétez-vous ces examens ? 483 La ferritine est à 1 102 μg/l (pour une normale entre A la radiographie du genou est en faveur de lésion 22 et 322 μg/l). Entraînement Quel(s) examen(s) doi(ven)t être réalisé(s) à ce stade d'arthrose pour rechercher des atteintes liées à la pathologie B la radiographie de genou est en faveur d'une retrouvée ? A échographie hépatique arthrite septique B dosage de la testostérone totale C la radiographie de genou est en faveur de lésion de C échographie cardiaque transthoracique D ostéodensitométrie chondrocalcinose E aucun examen car la ferritine n'est pas très élevée D l'ostéodensitométrie retrouve une ostéopénie E l'ostéodensitométrie retrouve une ostéoporose Question 12 Devant la ferritine élevée, un bilan de retentissement Question 14 de cette hémochromatose a été réalisé. L'échographie À quel stade de l'hémochromatose primitive est cardiaque retrouve une cardiopathie dilatée. L'ECG actuellement Monsieur V. ? ne retrouve pas de trouble du rythme. L'échographie A stade 0 abdominale est en faveur d'une cirrhose, sans lésion B stade 1 focale décelée évocatrice de carcinome hépatocellu- C stade 2 laire. La testostérone totale est à 1,3 ng/ml (N = 3-10). D stade 3 Pour interpréter ce taux de testostérone dans le cadre E stade 4 de l'hémochromatose, on peut s'attendre à : A hypogonadisme hypogonadotrope Question 15 B hypogonadisme hypergonadotrope Monsieur  V. se trouve au stade 4 de l'hémochro- C FSH et LH élevées matose, en raison du CS-Tf > 45  %, de la ferritine D FSH et LH normales ou basses élevée et de l'association à un diabète nécessitant un E prolactine élevée traitement par insuline, une probable cirrhose, une chondrocalcinose. Question 13 Quel(s) traitement(s) démarrez-vous dès à présent, en Le dosage des gonadotrophines est : dehors du traitement du diabète ? • FSH : 2 UI/l (N = 0,4-4) ; A traitement par PTH recombinante • LH : 0,4 UI/l (N = 0,4-3). B traitement par saignées Vous concluez à insuffisance gonadotrope. C traitement par testostérone La prostate est petite au toucher rectal. D traitement par gonadotrophines L'ostéodensitométrie retrouve un T-score à – 2,9 DS E traitement par vitamine D au niveau des cols fémoraux et – 3 DS au niveau lom- baire. La vitamine D est dosée à 12 mmol/l (N > 30). Question 16 Devant des douleurs articulaires, en particulier au Vous n'avez pas démarré le traitement par sai- niveau du genou gauche, qui est gonflé, vous avez gnées devant une contre-indication temporaire  : réalisé des radiographies dont voici un cliché. Hb < 11  g/dl. Vous avez commencé un traitement par testostérone devant son hypogonadisme hypo- gonadotrope probablement responsable de son ostéoporose. Vous l'avez également supplémenté en vitamine D. Il a réussi à se sevrer complètement en alcool et, quelques mois plus tard, son hémoglobine est mesu- rée à 12  g/dl. Il est toujours très fatigué. Un traite- ment par saignées est commencé. Il vous demande comment vont évoluer ses diffé- rentes atteintes à la suite des saignées. Vous pouvez envisager : A un sevrage de l'insulinothérapie B une réversibilité de la cirrhose C une amélioration de l'asthénie D une disparition de la mélanodermie E une normalisation de la fonction gonadotrope Cas clinique 31 Monsieur B., 38 ans, vous est adressé pour avis suite à la réalisation d'une IRM cérébrale il y a un mois dans le cadre de céphalées chroniques.

Cas cliniques Question 1 Question 2 L'image est donc évocatrice d'un craniopharyngiome. Quelle(s) est (sont) la (les) proposition(s) exacte(s) Quel(s) élément(s) clinique(s) allez-vous rechercher ? A des vergetures pourpres concernant cette IRM ? B un hypogonadisme A l'image « a » correspond à une coupe coronale en C une hémianopsie temporale D une baisse de l'acuité visuelle séquence T2 E un syndrome polyuro-polydipsique B l'image « b » correspond à une coupe coronale en Question 3 séquence T1 sans gadolinium Quelle est votre interprétation de ce champ visuel ? C la tige pituitaire est normale D l'hypophyse n'est pas visible E il existe une lésion solide suprasellaire 484 OG OD A hémianopsie bitemporale Question 5 B hémianopsie latérale homonyme Les résultats du bilan biologique initial sont les C simple élargissement la tache aveugle suivants : D quadranopsie bitemporale TSH < 0,01  mUI/l (N =  0,4-3,1) ; T4L  : 26,6  pmol/l E examen normal (N = 12,0-22,0) ; T3L : 8,8 (N = 2,9-4,9 pmol/l) ; ACTH (8 h) : 9 ng/l (N = 7-63 ng/l) ; cortisol (8 h) : 127 nmol/l Question 4 (N = 145-535 nmol/l) ; IGF-1 : 136 μg/l (88-246 μg/l) ; Quel(s) examen(s) biologique(s) prescrivez-vous en FSH : 2,1 UI/l (N = 1,1-7,2) ; LH : 0,84 UI/l (N = 1,3- première intention ? 5,8) ; testostérone : 2,5 nmol/l (N = 10,4-26) ; prolac- A TSHus tine : 22,5 μg/l (N = 4,0-15,2). B test au Synacthène® avec dosage du cortisol Quelle(s) est (sont) la (les) proposition(s) exacte(s) C IGF-1 concernant leur interprétation ? D LH, FSH, œstradiol A absence de déficit somatotrope E prolactine B hypothyroïdie centrale

Cas cliniques 25 C hypogonadisme central Quel(s) diagnostic(s) étiologique(s) retenez-vous 485 D diabète insipide finalement ? E hyperthyroïdie périphérique A cancer papillaire métastatique Entraînement Question 6 B maladie de Basedow Quel(s) signe(s) clinique(s) ou donnée(s) de l'interro- C prise cachée d'hormone thyroïdienne gatoire devez-vous rechercher pour orienter votre dia- D thyroïdite silencieuse gnostic étiologique d'hyperthyroïdie périphérique ? E surcharge iodée A une prise d'hormones thyroïdiennes B la réalisation d'un examen radiologique avec injec- Question 10 Quel(s) traitement(s) pouvez-vous proposer pour trai- tion de produit de contraste iodé ter son hyperthyroïdie secondaire à une thyroïdite C une exophtalmie silencieuse ? D un goitre A antithyroïdiens de synthèse E une perte de poids B diurétique Question 7 C bêtabloquant À l'interrogatoire, le patient vous décrit une perte D minéralocorticoïdes de poids récente de 5 kg, une nervosité inhabituelle E anxiolytiques mais qu'il met sur le compte de la découverte de cette lésion « cérébrale ». La thyroïde est plutôt atrophique. Question 11 Quel(s) examen(s) complémentaire(s) prescrivez-vous ? Le patient présente par ailleurs un syndrome poly- A la recherche d'anticorps anti-GAD uro-polydipsique avec une prise de boisson estimée à B une scintigraphie thyroïdienne au Tc99 5 litres par jour, évocateur d'un diabète insipide. C un dosage de la iodémie Quelle(s) est (sont) la (les) proposition(s) exacte(s) D un dosage des anticorps anti-TPO concernant ce diabète insipide ? E un PET-scan A il disparaîtra après chirurgie du craniopharyngiome Question 8 B il doit être confirmé par un test de restriction hydrique Le résultat de la scintigraphie est la suivante. C il n'y a pas de risque ionique si le patient boit « à sa soif » D il impose un régime pauvre en sel Concernant votre interprétation, quelle(s) est (sont) la E il doit être traité par hormone antidiurétique (les) proposition(s) exacte(s) ? A nodule basolobaire gauche extinctif (desmopressine) B scintigraphie blanche C fixation normale homogène de la thyroïde Question 12 D compatible avec une surcharge iodée Le patient a finalement été opéré de son craniopha- E compatible avec une thyroïdite ryngiome et, alors que les suites opératoires se pas- Question 9 saient bien, il présente trois jours après l'intervention Le bilan biologique complémentaire est le suivant : un syndrome confusionnel isolé ; son ionogramme est • thyroglobuline : 526,2 μg/l (N = 3,5-77 μg/l) ; le suivant : • anticorps anti-thyroglobuline : < 25 ; • sodium : 124 mmol/l (N = 136-145) ; • anticorps anti-TPO : 250 UI/l (N < 15). • potassium : 3,9 mmol/l (N = 3,4-4,5) ; • protéines : 65 g/l (N = 64-83) ; • urée : 4,4 mmol/l (N = 3,2-7,4) ; • créatinine : 61 μmol/l (N = 64-104) ; • glucose : 5,0 mmol/l. Quelles sont les causes possibles de cette hyponatré- mie ? A surdosage en hormone antidiurétique B hyperthyroïdie C insuffisance corticotrope D substitution insuffisante en hormone antidiurétique E sécrétion inappropriée d'hormone antidiurétique Question 13 Vous revoyez le patient un an après son intervention. Il va très bien. Son traitement est le suivant : • hydrocortisone 10 mg, un cp. le matin et ½ cp. à midi ; • L-thyroxine 125 μg, un cp. le matin à jeun ; • Androtardyl® (énanthate de testostérone) 250 mg, ½ ampoule tous les 15 jours ; • Minirinmelt® (desmopressine) 60 mg matin et midi et 120 mg le soir. Il vous montre son bilan biologique de contrôle réalisé un matin à jeun : • sodium : 138 mmol/l (N = 136-145) ;

Cas cliniques • potassium : 3,6 mmol/l (N = 3,4-4) ; Question 3 • TSH < 0,01 mUI/l (N = 0,4-3,1) ; Les résultats de votre bilan sont les suivants : • T4L : 18,6 pmol/l (N = 12,0-22,0) ; • FSH : 1,0 UI/l. • Cortisol à 8 h : 77 nmol/l (N = 145-535 nmol/l) ; – Valeurs plasmatiques chez la femme : • testostérone : 12,5 nmol/l (N = 10,4-26). – Phase folliculaire : 3,0 à 8,0 UI/l. Quelle(s) adaptation(s) thérapeutique(s) lui proposez- – Phase péri-ovulatoire : 2,5 à 16,7 UI/l. vous ? – Phase lutéale : 1,4 à 5,5 UI/l. A réduire la L-thyroxine à 100 μg par jour – Ménopause : 26,7 à 133,4 UI/l. B augmenter l'hydrocortisone à 20 mg le matin • LH : 0,03 UI/l. C maintenir l'Androtardyl® à l'identique – Valeurs plasmatiques chez la femme : D maintenir la L-thyroxine à 125 μg par jour – Phase folliculaire : 1,8 à 11,8 UI/l. E augmenter la L-thyroxine à 150 μg par jour – Pic ovulatoire : 7,6 à 89 UI/l. – Phase lutéale : 0,5 à 14 UI/l. Question 14 – Ménopause sans traitement hormonal subs- À la fin de la consultation, il vous fait part d'un projet de paternité. Que lui conseillez-vous ? titutif : 5 à 62 UI/l. A d'augmenter la fréquence des injections de testostérone • Prolactine : 62,6 μg/l. B de contrôler son spermogramme avant toute – Valeur sérique chez la femme en phase follicu- modification de traitement laire : 4,8 à 23,3 μg/l (médiane = 10,6 μg/l). C de remplacer la testostérone par la FSH recombinante D d'ajouter de la FSH recombinante à la testostérone • Testostérone : 0,52 nmol/l. E de remplacer la testostérone par de l'hCG et de la – Valeurs plasmatiques chez la femme  : 0,42 à 1,49 nmol/l. FSH recombinante • Œstradiol : < 17 pmol/l. – Valeurs plasmatiques chez la femme : Cas clinique 32 – Phase folliculaire : 92 à 367 pmol/l. – Phase lutéale  : 440 à 1 248  pmol/l (plateau Une patiente de 36 ans sans antécédent consulte pour thermique). un bilan d'infertilité primaire depuis 3 ans. Sur le plan – Pic préovulatoire : > 514 pmol/l. gynécologique, elle a été réglée spontanément à l'âge – Ménopause : < 73 pmol/l. 486 de 12 ans et ses cycles étaient réguliers jusqu'à la prise Comment interprétez-vous ce bilan ? d'une contraception orale œstroprogestative qu'elle a A hypogonadisme hypergonadotrophique arrêtée il y a 3 ans dans le cadre d'un projet de gros- B hyperprolactinémie sesse. Depuis l'arrêt de sa contraception, les cycles sont C hyperandrogénie irréguliers, toutes les 4 à 6  semaines. Ses dernières D hypothyroïdie centrale règles datent de 3 mois. La patiente décrit également E insuffisance gonadotrope l'apparition d'une acné modérée du visage. Question 4 À l'examen, vous notez : • poids 62 kg ; taille 1,65 m (IMC = 23) ; Quelle(s) proposition(s) est (sont) exacte(s) concernant • PA : 145/85 mm Hg ; • léger hirsutisme de la ligne ombilico-pubienne ; son hyperprolactinémie ? • thyroïde palpable homogène ; A elle peut être secondaire à un microadénome à • minime galactorrhée provoquée bilatérale. Le reste de l'examen clinique est sans particularité prolactine Votre patiente est infirmière, pratique une activité phy- B elle peut être secondaire à un macroadénome non sique modérée, fume 1 paquet par jour depuis 15 ans. fonctionnel Question 1 C elle peut être d'origine ovarienne Quel(s) élément(s) vous permet(tent) d'évoquer un D elle peut être secondaire à un macroadénome à prolactine E elle impose la réalisation d'un test à la TRH syndrome des ovaires polykystiques ? Question 5 A la présence d'une galactorrhée À la lecture de ce résultat, vous reprenez l'interroga- B l'indice de masse corporelle normal toire et vous apprenez que la patiente ronfle toutes C l'acné les nuits et que son mari a l'impression qu'elle s'arrête D l'hirsutisme parfois de respirer. Par ailleurs, elle ne peut plus porter E aucun élément son alliance et elle a pris deux pointures. Question 2 Quel(s) examen(s) complémentaire(s) demandez-vous ? Quels examens biologiques prescrivez-vous dans le A test à la dexaméthasone minute avec dosage du cadre du bilan d'infertilité ? cortisol plasmatique à 8 h A prolactinémie B dosage du cortisol et de la GH lors d'une épreuve B LH, FSH d'hypoglycémie insulinique C œstradiolémie C dosage du cortisol libre urinaire des 24 heures D testostérone totale D dosage de l'IGF-1 plasmatique E cortisol plasmatique à 8 h E dosage de la GH plasmatique à minuit

Cas cliniques 25 Question 6 Quelle(s) proposition(s) est (sont) exacte(s) ? 487 Dans le cadre de son bilan, une IRM hypophysaire a A il existe une compression chiasmatique été réalisée. B il existe un hypersignal T2 évocateur d'une nécrose hypophysaire C il persiste l'hypersignal physiologique de la posthypophyse D il s'agit d'un macroadénome hypophysaire E il existe une hypertension intracrânienne Question 7 À la lecture de cette IRM, quel(s) examen(s) complémentaire(s) devez-vous prescrire ? A dosage du cortisol lors d'un test au Synacthène® B exploration neuro-ophtalmologique avec acuité visuelle et champs visuels C test de restriction hydrique D examen ORL à la recherche d'une brèche ostéoméningée E scanner cérébral à la recherche d'une brèche ostéoméningée Question 8 Le champ visuel réalisé est le suivant. OG OD Entraînement Quelle est votre interprétation ? Quelle(s) mesure(s) thérapeutique(s) mettez-vous en A champ visuel normal place ? B quadranopsie bitemporale A mesures hygiéno-diététiques exclusive C hémianopsie bitemporale B mesures hygiéno-diététiques et metformine D hémianopsie latérale homonyme C mesures hygiéno-diététiques et sulfamide hypogly- E hémianopsie binasale cémiant Question 9 D indication de traitement médical de son acroméga- L'ensemble de votre bilan confirme votre diagnos- tic d'acromégalie. Quel bilan complémentaire réa- lie (par analogues de la somatostatine) en prépara- lisez-vous à la recherche de complications de son tion de la chirurgie acromégalie ? E insulinothérapie intensive avant prise en charge A un ECG chirurgicale de son acromégalie B une échographie cardiaque C une mesure de l'oxymétrie nocturne Question 11 D un dosage de la glycémie à jeun La patiente a été opérée par voie trans-sphénoïdale. E un coroscanner À J + 4 postopératoire, alors que la patiente est sous 30 mg d'hydrocortisone, elle présente des nausées et Question 10 des troubles de conscience. Son ionogramme est le Lors de son bilan préopératoire, vous notez une glycé- suivant : mie à jeun à 1,9 g/l et une HbA1c à 7,2 %. • sodium : 125 mmol/l (N = 136-145) ; • potassium : 3,9 mmol/l (N = 3,5-4,8).

Cas cliniques Sa diurèse est de 1 litre et sa PA à 130/80 mm Hg. Cas clinique 33 Quelle(s) proposition(s) thérapeutique(s) est (sont) Madame E., 31  ans, consulte pour aménorrhée persistant 6 mois après l'arrêt de sa contraception exacte(s) ? œstroprogestative. Les premières règles ont eu lieu A doubler les doses d'hydrocortisone à 14 ans ; les cycles ont ensuite toujours été régu- B ajouter de la fludrocortisone liers et la contraception a été prescrite à l'âge de C restriction hydrique à 500 ml par jour 21 ans. Elle ne prend pas de traitement, hormis des D apport en sel (3 g par jour) per os antalgiques 1 à 2 fois par mois pour céphalées, et E introduction d'hormone antidiurétique Question 12 des antiémétiques il y a une semaine pour un épi- L'épisode aigu postopératoire s'est rapidement résolu sode de nausées. après simple restriction hydrique. À l'examen, poids de 52  kg pour une taille de Vous revoyez la patiente un mois après cette interven- 164  cm, stable depuis au moins 5  ans ; fré- tion. La patiente va beaucoup mieux et vous décrit une quence cardiaque à 62/min., pression artérielle à sensation de désinfiltration du visage et des mains. 112/67 mm Hg. Elle a eu ses règles 2 semaines après l'intervention. Question 1 Quel bilan postopératoire proposez-vous pour évaluer Concernant les troubles des règles, quelle(s) sa fonction hypophysaire ? A test à la GnRH à la recherche d'un déficit affirmation(s) est (sont) exacte(s) ? A l'aménorrhée primaire est définie par l'absence de gonadotrope B test au Synacthène® à la recherche d'un déficit cycle menstruel après l'âge de 14 ans B la spanioménorrhée est définie par des règles corticotrope C TSHus et T4L abondantes et prolongées D LH, FSH, œstradiol C l'aménorrhée est physiologique dans les premiers E test d'hypoglycémie insulinique à la recherche d'un mois suivant l'arrêt d'une contraception œstropro- déficit somatotrope gestative D l'aménorrhée peut être liée à une carence nutri- Question 13 tionnelle La patiente souhaite remettre son projet de grossesse E des cycles menstruels réguliers témoignent du bon fonctionnement de l'axe gonadotrope 488 à plus tard et vous demande de lui prescrire une nou- Question 2 velle contraception orale. Quel bilan lui prescrivez-vous avant cela ? Quelle(s) est (sont) la (les) étiologie(s) compatible(s) A contrôle de la prolactinémie avec ce tableau ? B exploration d'une anomalie lipidique (cholestérol A adénome à prolactine total, triglycérides, HDLc, LDLc) B agénésie utérine C glycémie à jeun C grossesse D aucun, une contraception orale est contre-i­ndiquée D ménopause du fait de ses antécédents hypophysaires E aménorrhée hypothalamique fonctionnelle E contrôle de l'IGF-1 Question 3 Question 14 Le dosage d'hCG est négatif. Quel est votre bilan de Quel mode de contraception pouvez-vous lui proposer ? première intention ? A microprogestatif A LH B macroprogestatif B FSH C éthinyl-estradiol 20 μg et progestatif de troisième C œstradiol génération D prolactine D éthinyl-œstradiol 50 μg et progestatif de première E progestérone génération Question 4 E dispositif intra-utérin Les résultats sont les suivants : Question 15 • LH : 3 UI/l (N = 2-5) ; Vous avez prescrit une contraception par micropro- • FSH 2 UI/l (N = 2-8) ; gestatif continue (Cerazette®). • œstradiol  : 15  pg/ml (norme en début de phase Quelle(s) est (sont) la (les) proposition(s) exacte(s) ? folliculaire = 10-50) ; A elle doit être prise tous les jours à heure fixe • prolactine : 78 ng/ml (N = 11-25). B en cas d'oubli, elle peut être prise dans les Quel(s) est (sont) les diagnostics(s) compatible(s) avec 12 heures sans adaptation ce bilan ? C en cas d'oubli de plus de 12 heures, une contra- A une grossesse débutante ception mécanique doit être maintenue pendant B une insuffisance ovarienne un mois C un microadénome à prolactine D l'aménorrhée est rare D un craniopharyngiome E elle agit exclusivement au niveau utérin E l'antécédent de prise de pilule

Cas cliniques 25 Question 5 Question 8 489 L'IRM hypophysaire est présentée ci-dessous. Elle vous demande quels sont les composés conte- nus dans les pilules œstroprogestatives de deuxième génération. Que lui répondez-vous ? A gestodène B éthinyl-œstradiol C estrone D progestérone E lévonorgestrel Quelle(s) proposition(s) est (sont) exacte(s) ? Question 9 Entraînement A il s'agit d'une coupe en T1 Avant de prescrire une contraception œstroproges- B la lésion est compatible avec un macroadénome tative, selon les recommandations de la HAS, il est demandé de réaliser : hypophysaire A un dosage du cholestérol total C la lésion est compatible avec un microadénome à B un dosage des triglycérides C un dosage de la glycémie à jeun prolactine D un frottis cervico-vaginal D la lésion a une extension dans le sinus caverneux E une mesure de la pression artérielle gauche Question 10 E la lésion a une extension suprasellaire Deux  ans plus tard, elle revient vous voir en consul- tation : elle a arrêté la contraception car elle a avec Question 6 son compagnon un projet parental. Les cycles sont Quels mécanismes peuvent expliquer l'aménorrhée réguliers. Elle s'inquiète de sa fertilité. chez cette jeune fille dans ce contexte ? Quelle(s) proposition(s) est (sont) exacte(s) ? A action de l'hyperprolactinémie sur la GnRH A la fertilité de la femme diminue après 35 ans B action de l'hyperprolactinémie sur la croissance B l'exploration du couple est justifiée après 1 an de folliculaire rapports sexuels non protégés C action de l'hyperprolactinémie sur l'utérus C la fertilité d'un couple dépend de la fécondité de D destruction des cellules gonadotropes par la tumeur E action de l'hyperprolactinémie sur la glande mammaire l'un et de l'autre partenaire D dans la population générale, 50  % des gros- Question 7 Vous traitez avec succès la patiente par agonistes sesses souhaitées sont obtenues après 6  mois dopaminergiques. d'essai Deux ans plus tard, le traitement peut être arrêté, la E la prise en charge de l'infertilité s'arrête à la date lésion hypophysaire a disparu. Madame E. a retrouvé anniversaire des 40 ans de la femme des cycles menstruels réguliers. Elle a un nouveau partenaire et souhaite reprendre une contraception Question 11 œstroprogestative, qui lui convenait parfaitement. Quel(s) examen(s) allez-vous prescrire à Madame E. Quelle(s) recommandation(s) lui faites-vous ? en première intention dans le cadre de l'exploration A la pilule doit être commencée au premier jour des de l'infertilité ? A une hystérosalpingographie règles B une échographie pelvienne B la pilule doit être prise durant un repas C une hystéroscopie C la pilule doit être prise à heure fixe D un bilan hormonal au 14e jour du cycle D en cas d'oubli de moins de 12  heures, il faut E une courbe ménothermique prendre la pilule oubliée puis poursuivre la pla- Question 12 quette habituellement Son conjoint est âgé de 40 ans et n'a jamais eu d'en- E en cas d'oubli de plus de 12 heures, il faut se protéger fant auparavant. par une contraception mécanique pendant 7 jours À propos du spermogramme, quelle(s) proposition(s) est (sont) exacte(s) ? A il doit être réalisé avec un délai d'abstinence de 3 à 5 jours B il doit être réalisé à jeun C en cas d'anomalie, il doit être répété à distance pour confirmer celle-ci D la tératospermie correspond à une diminution de la vitalité des spermatozoïdes E l'aspermie correspond à une absence de spermato- zoïdes

Cas cliniques Question 13 C vous demandez une ponction cytologique du Le couple finit par concevoir avant la fin du bilan pres- nodule crit. Madame E. accouche d'une petite fille en bonne D vous demandez un dosage de thyroglobuline santé. Le suivi est marqué par la survenue d'une frac- E vous demandez une scintigraphie thyroïdienne ture du poignet lors d'une chute à 42 ans. À 48 ans, Question 2 elle commence à se plaindre de bouffées de chaleur Vous demandez une cytoponction thyroïdienne associée à une irrégularité menstruelle environ 3 à qui retrouve des atypies cellulaires. Le prélèvement 4 cycles par an. est classé en catégorie  5 selon la classification de Quelle(s) est (sont) la (les) caractéristique(s) des bouf- Bethesda (risque de malignité de 60 à 70  %). Une fées de chaleur liées à la ménopause ? thyroïdectomie totale associée à un curage ganglion- A association à des céphalées et des palpitations B sensation de chaleur suivie de sueurs naire central est réalisée. C déclenchement par les repas D déclenchement par l'effort Quels sont les risques spécifiques de l'opération ? E recrudescence nocturne A paralysie récurrentielle uni- ou bilatérale B paralysie faciale C hématome suffocant Question 14 D hyponatrémie Madame E. a maintenant une aménorrhée depuis E hypoparathyroïdie 12 mois, toujours associée à des bouffées de chaleur rendant son sommeil difficile. Question 3 Quelle(s) est (sont) l'(es) indication(s) à proposer un Un traitement substitutif de L-T4 est mis en place en traitement hormonal de la ménopause chez cette postopératoire. patiente ? A l'intensité des bouffées de chaleur Quelle(s) proposition(s) est (sont) exacte(s) ? B l'antécédent de fracture A le traitement est commencé après avis cardiolo- C la pauciparité D l'âge gique E l'antécédent de contraception hormonale B le traitement est commencé d'emblée à pleine dose C ce traitement sera définitif 490 Question 15 D sa dose est adaptée sur le dosage de TSH Vous prescrivez un traitement hormonal de la méno- E sa dose est adaptée sur le dosage de thyroglobu- pause. Comment allez-vous adapter la dose d'œstro- line gènes chez votre patiente ? Question 4 A une tension mammaire est un signe de surdosage Le lendemain de l'intervention, la patiente se plaint en œstrogènes B un dosage d'œstradiol bas nécessite une augmen- de paresthésies des membres supérieurs et de tation des doses employées crampes douloureuses. La calcémie est à 1,6 mmol/l C un T-score bas à l'ostéodensitométrie justifie une (N = 2,2-2,6). augmentation des doses D un taux élevé de FSH nécessite une augmentation Quelle est votre attitude ? A surveiller et recontrôler des doses B donner du calcium E une persistance des bouffées de chaleur nécessite C donner du magnésium D donner de la vitamine D une augmentation des doses E donner de la PTH Cas clinique 34 Question 5 Sous supplémentation vitaminocalcique, le contrôle Madame P., âgée de 52 ans, aux antécédents de can- biologique effectué un mois plus tard retrouve une cer du sein, opéré 2  ans plus tôt et traité par anti- calcémie à 2,9 mmol/l (N = 2,2-2,6). aromatases, bénéficie dans le cadre de son suivi d'une A vous arrêtez la supplémentation calcique scintigraphie PET-scan au 18FDG qui ne montre pas B vous augmentez la dose de vitamine D de foyer de fixation pathologique, en dehors d'une C vous mettez en place une restriction hydrique fixation cervicale intense correspondant à un nodule D vous faites un ECG thyroïdien du lobe thyroïdien droit de 42 mm de dia- E vous perfusez des bisphosphonates mètre. La fonction thyroïdienne est normale avec une TSH à 3 mUI/l (N = 0,5-5). Question 6 L'examen anatomopathologique retrouve un carci- Question 1 nome thyroïdien papillaire de 41 mm de diamètre, Face à ce nodule thyroïdien fixant au PET-scan : avec extension extrathyroïdienne débutante et A vous craignez une métastase du cancer du sein envahissement de 6 sur 14  ganglions, de stade B vous craignez un cancer thyroïdien pT3N1R0.

Cas cliniques 25 Quels sont, chez cette patiente, les facteurs de risque Question 11 491 Quelques mois plus tard, Madame P. se plaint d'une de récidive ? lombalgie aiguë, résistante aux traitements conven- A âge > 50 ans tionnels. La thyroglobuline est à 350 μg/l (N < 1). Une B sexe féminin IRM thoracolombaire est pratiquée. C volume du nodule D histologie papillaire E extension ganglionnaire Question 7 Que vous montre cet examen ? Entraînement Compte tenu de ces éléments, on prévoit de complé- A masse tumorale en L2 ter le traitement chirurgical par une dose de 100 mCi B recul du mur postérieur (3 700 MBq) d'iode radioactif. C lésion en hypersignal T1 Concernant ce traitement, quelle(s) proposition(s) est D ostéomalacie (sont) exacte(s) ? E ostéopénie A il peut être réalisé en externe, sans hospitalisation B il justifie une hospitalisation en chambre Question 12 Il faut différencier une métastase du cancer thyroïdien radioprotégée d'une métastase du cancer du sein. C il va nécessiter une radioprotection de l'entourage Quels sont les arguments pour une origine thyroï- D il fait courir un risque d'aplasie médullaire dienne de la lésion ? E il fait courir un risque de stérilité chez la femme A élévation de la thyroglobuline plasmatique B caractère ostéolytique de la lésion jeune C fixation de la lésion au PET-scan D fixation de la lésion à l'iode radioactif Question 8 E présence d'un immunomarquage à la thyroglobu- Le traitement isotopique est administré dans un service de médecine nucléaire. La scintigra- line sur la métastase phie corporelle totale retrouve un résidu cervical exclusif. La L-thyroxine est prescrite à la dose de Question 13 125  μg par jour. La calcémie est normalisée sans À l'interrogatoire et l'examen clinique, que pourriez- traitement. vous retrouver ? Quels sont les examens utiles au suivi ? A diminution de la force musculaire des membres A dosage de thyroglobuline plasmatique B dosage de la TSH inférieurs C scintigraphie osseuse B réflexes vifs polycinétiques D dosage de la calcitonine C aréflexie des membres inférieurs E échographie cervicale D trouble de la sensibilité des membres inférieurs E incontinence urinaire Question 9 Si le traitement de L-thyroxine n'est pas pris régulière- Question 14 ment, quelles seront les conséquences ? La métastase du cancer thyroïdien est confirmée. La A dyspnée patiente subit une métastasectomie avec vertébroplas- B hypertension artérielle tie puis administration d'une dose d'iode radioactif. C prise de poids Quelques semaines plus tard, sous traitement de L-T4, D hypercholestérolémie le bilan thyroïdien est le suivant : E risque de récidive du cancer • TSH : 0,01 mUI/l (N = 0,4-4) ; • T3 : 5,9 pmol/l (N = 3,2-5,2) ; Question 10 Trois mois plus tard, vous la revoyez en consulta- tion avec le bilan suivant sous 125  μg par jour de lévothyroxine : • TSH =  6,4  mUI/l (objectif  : 0,1-0,5  mUI/l) ; T4L = 14 pmol/l (N = 10-22) ; • calcémie = 2,23 mmol/l (N = 2,2-2,6) ; phosphoré- mie = 0,89 mmol/l (N = 0,75-0,95). Quelle(s) proposition(s) est (sont) exacte(s) ? A vous contrôlez le bilan avant de modifier le traitement B vous augmentez la dose de L-thyroxine C vous diminuez la dose de L-thyroxine D vous prévoyez un contrôle du bilan thyroïdien 15 jours après modification thérapeutique E vous prévoyez un contrôle du bilan thyroïdien 6 semaines après modification thérapeutique

Cas cliniques • T4L : 24 pmol/l (N = 12-22) ; Question 2 • thyroglobuline : 60 μg/l (N < 1). La cytoponction thyroïdienne retrouve des atypies La patiente se plaint de bouffées de chaleur et d'accès cellulaires. Elle est classée en catégorie 5 selon la clas- d'hypersudation. sification de Bethesda. Une intervention chirurgicale Quelle(s) cause(s) peut-on évoquer ? A syndrome climatérique de la ménopause est programmée. B surdosage en L-T4 C manifestations secondaires du traitement anti-­ Quel(s) examen(s) faut-il demander en préopératoire ? A dosage de thyroglobuline aromatase B dosage des anticorps anti-thyroglobuline D métastase hypophysaire du cancer thyroïdien C scanner cervicothoracique E hypersécrétion de thyroglobuline D scintigraphie thyroïdienne à l'iode 131 E bilan de coagulation préopératoire standard Question 15 Question 3 Dans le cadre du bilan d'extension, vous demandez un scanner abdominal. Une thyroïdectomie totale associée à un curage gan- glionnaire central est réalisée. Quels sont, parmi ces propositions, les risques spéci- fiques de l'opération ? A paralysie récurentielle uni- ou bilatérale B paralysie faciale C surdité D hématome suffocant E hypothyroïdie Question 4 Un traitement substitutif de L-T4 est mis en place en postopératoire. Quelle(s) est (sont) la (les) proposition(s) exacte(s) ? A le traitement est commencé à dose progressive B le traitement est commencé d'emblée à pleine dose C le traitement est commencé sous surveillance ECG 492 D le traitement est commencé après avis cardiologique E le traitement sera définitif Que montre cette imagerie ? Question 5 A lésion surrénalienne gauche B lésion surrénalienne bilatérale Le lendemain de l'intervention, le patient se plaint de C tumeur du côlon D refoulement du rein gauche paresthésies des membres supérieurs et de crampes E volumineuses métastases hépatiques douloureuses. Quel(s) dosage(s) proposez-vous ? A calcémie B kaliémie C phosphorémie D natrémie E TSH Cas clinique 35 Question 6 Benjamin, âgé de 17  ans, consulte pour un nodule L'examen anatomopathologique retrouve un car- thyroïdien, découvert en faisant sa toilette. Il a été traité par radiothérapie et chimiothérapie à l'âge de cinome thyroïdien papillaire unifocal, réséqué en 9 ans pour un médulloblastome du cervelet. La palpation cervicale retrouve un nodule ferme d'envi- totalité, sans extension extrathyroïdienne ni ganglion- ron 2 cm de diamètre du lobe gauche sur thyroïde pal- pable. L'échographie confirme l'existence d'un nodule naire, de 22 mm de diamètre. unique, solide, hypervasculaire, hypoéchogène, hété- rogène, de 21 mm de diamètre au sein d'une thyroïde Quelle(s) est (sont) la (les) proposition(s) exacte(s) ? homogène. La fonction thyroïdienne est normale. A le pronostic est mauvais B un suivi prolongé sera nécessaire C le marqueur de suivi du cancer sera la thyroglobu- line plasmatique D le marqueur de suivi du cancer sera la calcitonine E ce cancer est hormonodépendant vis-à-vis de la TSH Question 1 Question 7 Quels sont les éléments qui vous conduisent à deman- Si le traitement de L-thyroxine n'est pas pris réguliè- der une cytoponction ? rement, quelle(s) conséquence(s) peut-on craindre ? A taille du nodule A prise de poids B normalité de la concentration plasmatique de TSH B hypercholestérolémie C antécédent de radiothérapie C constipation D antécédent de chimiothérapie D acné E antécédent de cancer E tachycardie

Cas cliniques 25 Question 8 Quels sont les signes cliniques et biologiques compa- 493 Trois mois plus tard, vous revoyez Benjamin en consul- tibles avec le diagnostic ? tation avec le bilan suivant sous 125 μg par jour de A hypotension Entraînement lévothyroxine : B hyponatrémie • TSH =  6,4  mUI/l (objectif  : 0,1-0,5  mUI/l) ; T4L C hyperkaliémie D hypoglycémie = 14 pmol/l (N = 10-22) ; E mélanodermie • calcémie = 2,23 mmol/l (N = 2,2-2,6) ; phosphoré- Question 14 mie = 0,89 mmol/l (N = 0,75-0,95). Quels sont les résultats attendus du bilan hormonal ? Que faites-vous ? A cortisol plasmatique à 8 h abaissé A vous augmentez la dose de lévothyroxine B ACTH plasmatique à 8 h augmentée B vous diminuez la dose de lévothyroxine C bonne réponse du cortisol au test au Synacthène® C vous contrôlez le bilan avant de modifier le traitement D rénine basse D vous prévoyez un contrôle du bilan thyroïdien E aldostérone basse 15 jours après modification thérapeutique Question 15 E vous prévoyez un contrôle du bilan thyroïdien Le diagnostic d'insuffisance corticotrope est confirmé et un traitement par hydrocortisone 20  mg par jour 6 semaines après modification thérapeutique en continu est prescrit. Quelle(s) est (sont) la (les) proposition(s) exacte(s) ? Question 9 A on recommande un régime pauvre en sel Après adaptation de son traitement, Benjamin se sent B on ajoute un diurétique pour éviter une prise de poids bien mais note l'apparition d'une gynécomastie. C il faudra adapter le traitement en cas de stress Parmi les propositions suivantes, quel(s) dosage(s) D il faut prévoir une éducation thérapeutique demandez-vous en première intention ? E on lui recommande le port d'une carte d'insuffi- A prolactine B testostérone sant surrénalien C FSH D thyroglobuline Cas clinique 36 E AMH Un patient de 22  ans consulte pour une asthénie, Question 10 avec une insomnie, des palpitations et des céphalées Le bilan hormonal de débrouillage est le suivant : FSH depuis 7 mois. Il est trader, marié, sans enfant. Il n'a = 56 UI/l (N = 2-7), testostérone basse. pas d'antécédent particulier hormis une appendicite Quelle est votre interprétation ? La gynécomastie est à l'âge de 20 ans, un burn-out l'année précédente et secondaire à : une constipation opiniâtre. A une insuffisance gonadotrope L'interrogatoire trouve : B une insuffisance gonadique primitive • pas de traitement ; C une insuffisance antéhypophysaire • tabagisme actif ; D une inertie hypothalamique • antécédents familiaux de diabète chez sa mère, E un microprolactinome d'HTA chez son père et son grand-père maternel. Question 11 L'examen clinique objecive : L'insuffisance testiculaire primitive est confirmée. • poids 75 kg (poids habituel 80 kg) ; taille 1,77 m ; Benjamin vous en demande la cause. • PA : 150/85 mm Hg ; FC : 109 bat./min., régulière ; Parmi les propositions suivantes, quelle(s) est (sont) la (les) réponse(s) exacte(s) ? auscultation cardiopulmonaire : tachycardie ; A conséquence de la radiothérapie • pas d'œdème des membres inférieurs ; B conséquence de l'hypothyroïdie • abdomen souple ; sensibilité de la fosse lombaire C conséquence de la chimiothérapie D atteinte transitoire gauche. E atteinte définitive Un holter tensionnel des 24  heures est réalisé, confirmant l'hypertension artérielle avec une tension Question 12 moyenne de 160/90  mm  Hg et trois pics durant la Sur le plan pratique, que proposez-vous ? journée au-dessus de 180/100 mm Hg. A traitement substitutif androgénique B induction de la spermatogenèse par des injections Question 1 Devant ce tableau clinique, vous évoquez une HTA. de gonadotrophines Parmi les éléments suivants, lesquels vous orientent C contraception efficace pour sa compagne vers une HTA secondaire ? D don d'ovocyte pour obtenir une grossesse A âge du patient E don de sperme pour obtenir une grossesse B asthénie associée C consommation de réglisse Question 13 D HTA associée à une hypokaliémie Il revient quelques mois plus tard et se plaint d'être E antécédents familiaux de coronaropathie à 60 ans fatigué. Vous craignez une insuffisance corticotrope du fait de la radiothérapie cérébrale.

Cas cliniques Question 2 Question 5 Quelles endocrinopathies peuvent être associées à une HTA ? Le premier bilan d'hypertension est normal. A acromégalie B hypothyroïdie Après avoir réinterrogé le patient, il vous apprend que C adénome de Conn D bloc en 21-hydroxylase lors de son appendicite 10 ans auparavant, un scanner E phéochromocytome aurait été réalisé montrant un nodule de la surrénale. Question 3 Parmi les éléments suivants, lesquels plaideraient en Lorsque vous lui demandez comment s'était passée faveur d'un hyperaldostéronisme primaire ? A hypokaliémie l'anesthésie, il vous répond qu'il avait eu des petits B hypernatrémie C rénine plasmatique augmentée soucis de tension mais ne se souvient plus exactement D cortisol libre urinaire augmenté E kaliurèse basse de quoi il s'agissait. Devant ce tableau, vous évoquez le diagnostic de phéo- chromocytome même s'il s'agit d'une pathologie rare. Quel bilan complémentaire demandez-vous ? A dosage des catécholamines plasmatiques B dosage des métanéphrines urinaires C dosage de l'aldostérone plasmatique D dosage de la SDHEA plasmatique E dosage de la sérotonine Question 4 Question 6 • Bilan biologique : kaliémie : 3,8 mmol/l (N = 3,5- Le bilan hormonal est en faveur d'un phéochromo- 5) ; natrémie : 139 mmol/l (N = 135-145) ; glycémie à jeun : 1,10 g/l (N = 0,70-1,10 g/l). cytome. Quel bilan complémentaire demandez-vous ? • Bilan lipidique : normal. A échographie abdominale • BU : normale. B cathétérisme des veines surrénaliennes • ECG : quelques extrasystoles auriculaires. C scanner surrénalien Parmi les éléments suivants, lesquels plaideraient en D scintigraphie au MIBI E scintigraphie au noriodocholestérol faveur d'un phéochromocytome ? Question 7 A hypotension alternant avec l'HTA Les métanéphrines plasmatiques sont augmentées à 494 B diabète 7 fois la normale. C hirsutisme Le scanner (a) et le PET-scan au 18FDG (b) montrent une D diarrhée lésion surrénalienne gauche de 55  mm. La lésion est E céphalées hypermétabolique lors de la scintigraphie à la MIBG (c). a bc Vous retenez donc le diagnostic de phéochromocy- B néoplasie endocrinienne multiple de type 1 tome gauche. Quelle prise en charge proposez-vous ? C syndrome de Klinefelter A surveillance D néoplasie endocrinienne multiple de type 2 B traitement par spironolactone E syndrome de Turner C traitement par surrénalectomie D préparation préopératoire par bêtabloquants Question 9 E traitement par chimiothérapie Parmi les éléments suivants, lesquels auraient pu vous faire suspecter un syndrome de Cushing ? Question 8 A vergetures pourpres Parmi les maladies génétiques suivantes, lesquelles B « bosse de bison » sont susceptibles de causer un phéochromocytome ? C exophtalmie A mutation du gène codant les sous-unités de l'en- D céphalées, sueurs et palpitations E érythrose faciale zyme succinate déshydrogénase

Cas cliniques 25 Question 10 volume ovarien augmenté > 10 ml. 495 Parmi les éléments suivants, lequel (lesquels) est (sont) utile(s) pour dépister le syndrome de Cushing ? Question 8 A cortisol à 8 h Réponse : C, E. B ACTH Commentaire : Il est favorisé par un surpoids ou une C cortisol à 0 h obésité, source d'insulinorésistance ; c'est une ano- D test au Synacthène® malie ovarienne primitive. E test de freinage minute à la dexaméthasone Question 9 Réponses Réponse : A, B, E. Avis au lecteur  : certains cas cliniques n'ont pas de Question 10 pondérations, de PMZ ou de SMZ. Réponse : B, C, D. Commentaire : Il n'apparaît justifié de commencer l'ex- Cas clinique 1 ploration d'un couple souhaitant un enfant qu'après une année de rapports sexuels sans contraception. Une Question 1 exploration plus rapide peut être engagée si : Réponse : C. • la femme a plus de 35 ans ; Commentaire : Aménorrhée secondaire = Grossesse, • les troubles du cycle sont patents ; quel que soit le contexte. • une pathologie de l'appareil génital est connue ou Question 2 suspectée chez l'homme ou la femme. Réponse : A, D, E. Chez la femme de plus de 45  ans, il ne paraît pas • A : Le syndrome de Turner peut se présenter sous forme souhaitable de commencer des explorations pour un bilan de stérilité. d'une aménorrhée secondaire chez une patiente ayant eu un développement pubertaire complet. Grille d'évaluation • B, C : absence d'utérus donc cause d'aménorrhée primaire. No Réponses Points PMZ SMZ 1C 5 C C Question 3 2 A, D, E 15 (5, 5, 5) D Réponse : A, B, E. 3 A, B, E 10 (3, 4, 3) Entraînement Commentaire : 4 A, B, C, D 10 (3, 3, 2, 2) • C : Le craniopharyngiome se présente sous forme 5 B, E 10 (5, 5) 6 A, B, C 10 (3, 4, 3) d'une insuffisance gonadotrope sans hyperandro- 7B 10 génie. 8 C, E 10 (5, 5) • D : L'aménorrhée fonctionnelle post-pilule n'existe pas. 9 A, B, E 10 (4, 3, 3) 10 B, C, D 10 (4, 3, 3) Question 4 100 Réponse : A, B, C, D. Total Commentaire : Bilan de première intention devant ce tableau (la prolactine en fait aussi partie). Cas clinique 2 Question 5 Question 1 Réponse : B, E. Réponse : C. Commentaire  : L'aménorrhée est primaire ; il ne Question 6 s'agit pas d'un impubérisme car le développement Réponse : A, B, C. mammaire a débuté. Les items D et  E ne peuvent Commentaire  : L'obésité est souvent associée mais être retenus a priori par principe (diagnostics « d'éli- inconstante, et n'est pas un critère diagnostique. mination »), en particulier dans le cas de cette patiente qui présente des anomalies anamnestiques Question 7 et cliniques. Réponse : B. Commentaire  : La classification dite de Rotterdam Question 2 doit être utilisée ; elle repose sur trois critères : Réponse : A, B, D. • existence d'une anomalie du cycle menstruel  : Commentaire : Les items C et E ne sont pas des exa- mens à visée étiologique d'une aménorrhée. aménorrhée, spanioménorrhée ou cycles longs ; • existence d'une hyperandrogénie clinique et/ou biologique ; • présence d'ovaires polykystiques à l'échogra- phie : présence d'au moins 12 follicules mesurant 2-9  mm de diamètre dans chaque ovaire et/ou

Cas cliniques Question 3 Question 9 Réponse : C. Réponse : A, C, D, E. Commentaire  : Les items A, D, E mettraient en évi- Commentaire : dence un œstradiol bas sans élévation de la FSH, • A : Il faut maintenir le traitement substitutif (THS) l'item B un œstradiol élevé et une FSH très basse. jusqu'au début de la grossesse. Dans ce contexte Question 4 de désir de grossesse, un THS peut être plus facile à Réponse : B. accepter qu'un contraceptif œstroprogestatif pour Commentaire : la patiente. • A : L'IRM hypophysaire ne fait pas partie du bilan • B  : Le citrate de clomifène est inefficace en cas de carence œstrogénique (anovulation de des hypogonadismes hypergonadotropes. classe III de l'OMS dans le cas de Clémence, qui • C  : L'échographie pelvienne permettrait de mon- n'entre pas dans les indications du citrate de clomifène). trer l'atrophie ovarienne mais elle ne vient pas en • C : La grossesse sera a priori obtenue par don. premier dans le diagnostic étiologique. • D  : Il est impératif de refaire un bilan cardiolo- • D : La radiographie de la main contribue à préciser gique avant la grossesse (recommandations du le statut pubertaire. CNGOF, PNDS Syndrome de Turner), avec IRM en • E : L'ostéodensitométrie évalue l'impact osseux de général. la carence œstrogénique. • E : La TSH n'est pas dans l'objectif pour une femme Cf. Programme national de diagnostic et de soin désirant une grossesse (< 2,5 mUI/l). (PNDS) du syndrome de Turner (HAS, 2008). Question 5 Question 10 Réponse : Syndrome de Turner. Oui, dans 50 % des Réponse : B. cas (les autres formules comportent les mosaïques, les Commentaire : anomalies de structure de l'X, etc.). • A : Le suivi de ce type de grossesse à risque relève Question 6 d'une maternité de type 2 ou 3 selon les patholo- Réponse : A, B, C. gies associées (cf. PNDS). • A, B  : Les maladies auto-immunes sont significa- • B : Le dépistage ciblé au premier trimestre est indi- 496 tivement plus fréquentes en cas de syndrome de qué en raison du risque accru de diabète de type 1 mais aussi de type 2 chez ces patientes. Turner, dont les endocrinopathies auto-immunes • C : L'acide folique est prescrit à 0,4 mg par jour (diabète de type  1 et maladie de Hashimoto), la en absence d'indication spécifique au dosage à maladie cœliaque qui doit être dépistée. 5 mg. • C : La surdité est la conséquence des otites chro- • D : Il n'y a pas d'indication au DAN en cas de don niques ; un suivi ORL est nécessaire ; même limitée d'ovocyte. à une hypoacousie elle contribue au retard scolaire. • E : Les césariennes sont plus fréquentes (dispropor- • D, E : Le cancer du sein et le mélanome ne sont pas tion maternofœtale) mais non systématiques. plus fréquents, y compris en cas de nævus multiples. Question 7 Grille d'évaluation Points Réponse : A, B, D. 10 Commentaire : Les examens demandés dépistent une No Réponses 10 malformation rénale, une malformation cardiaque et 1C 10 aortique, une maladie cœliaque. La fibroscopie n'in- 2 A, B, D 10 tervient qu'en cas de suspicion clinique ou biologique 3C 10 de maladie cœliaque et le dosage de créatinine en cas 4B de malformation rénale. 5 Syndrome de 10 10 Question 8 Turner. Oui 10 Réponse : A, B. 6 A, B, C 10 Commentaire : 7 A, B, D 10 • B : Le traitement peut entraîner une soudure pré- 8 A, B 100 9 A, C, D, E maturée des cartilages de conjugaison s'il n'est pas 10 B conduit de façon appropriée. • C : Il s'agit d'un traitement préventif de complica- Total tions cardiovasculaires et osseuses graves. • D : Il est proposé jusqu'à 50 ans en l'absence de contre-indication. • E  : Le bilan d'hémostase n'est pas effectué, sauf point d'appel anamnestique ou clinique.

Cas clinique 3 Cas cliniques 25 Question 1 Cas clinique 4 497 Réponse : A, B, C, E. Question 1 Entraînement Question 2 Réponse : C (PMZ). Réponse : B, D. Commentaire  : Il doit être réalisé en présence d'un des cinq facteurs de risque des recommandations. Question 3 Réponse : A, B, D. Question 2 Réponse : A, B. Question 4 Commentaire : Réponse : C. • C : Le seuil rénal de réabsorption du glucose est dimi- Question 5 nué pendant la grossesse. De ce fait, la glycosurie n'est Réponse : C, D. pas utilisable comme outil de dépistage/sélection. • D : Seul le dosage de la glycémie à jeun est recom- Question 6 mandé à ce stade. Réponse : B. • E : D'après la HAS, l'HbA1c ne doit pas être utilisée comme un outil diagnostique des états hypergly- Question 7 cémiques. En outre, son interprétation pendant la Réponse : B. grossesse est difficile en raison de l'hémodilution. Question 8 Question 3 Réponse : A, E. Réponse : B (PMZ). Commentaire  : Le seuil pathologique est ≥ 0,92  g/l Question 9 (diabète gestationnel précoce). On parle de diabète Réponse : C, E. avéré découvert pendant la grossesse en cas de gly- cémie ≥ 1,26 g/l. Question 10 Réponse : C. Question 4 Réponse : A (PMZ). Question 11 Commentaire : Un nouveau dépistage par charge de Réponse : A. 75 g de glucose est nécessaire car : • elle présente un des cinq facteurs de risque ; Question 12 • son dépistage de début de grossesse était normal Réponse : B, C, D. (au cas échéant, la patiente aurait été prise en Grille d'évaluation Points charge dès le dépistage de début de grossesse) ; 10 • on observe une augmentation de l'insulinorésis- No Réponses 8 tance à ce terme. 1 A, B, C, E 9 2 B, D 8 Question 5 3 A, B, D 8 Réponse : C (PMZ). 4C 8 Commentaire  : Il suffit d'une seule valeur anormale 5 C, D 8 pour conclure à un diabète gestationnel : 6B 8 • glycémie à jeun 0,92-1,25 g/l ; 7B 8 • glycémie 1 heure post-charge 1,80 et plus ; 8 A, E 8 • glycémie 2 heures post-charge 1,53-1,99 g/l. 9 C, E 8 On définit la dysglycémie comme un diabète avéré 10 C 9 découvert pendant la grossesse si la glycémie à jeun 11 A 100 est ≥ 1,26  g/l ou la glycémie 2  heures post-charge 12 B, C, D est ≥ 2,0 g/l (les seuils diagnostiques en dehors de la grossesse), mais il n'y a pas de norme pour la glycémie Total 1 heure post-charge. Question 6 Réponse : A, D. Commentaire : • B  : Pas de malformations congénitales, qui sur- viennent pendant l'organogenèse (8  premières semaines de grossesses). Pour mémoire, la glycé- mie à jeun était normale en début de grossesse. • E  : Risque d'hypoglycémie néonatale mais pas de diabète néonatal (hyperinsulinisme fœtal per- sistant 48 heures si le diabète maternel n'est pas contrôlé les jours précédant la grossesse).

Cas cliniques Question 7 l'hyperglycémie modérée à jeun (glycémie à jeun  : Réponse : C (PMZ). 1,10-1,25  g/l) et l'intolérance au glucose (glycémie Commentaire : à 2 heures : 1,40-1,99 g/l). Un dépistage avant une • A  : L'activité physique (30  minutes 3 à 5 fois prochaine grossesse est recommandé mais il n'y a pas par semaine) est recommandée en l'absence de de contre-indication. contre-indication obstétricale. • B : Ne pas diminuer les apports caloriques en des- Question 11 sous de 1 600 kcal par jour. Réponse : B, D, E. • D  : Attention  : glucides et lipides sont particuliè- Commentaire  : Seule l'insuline peut être utilisée rement importants (+++)  : ne pas trop réduire pendant la grossesse ; il n'y a cependant pas d'effets les apports glucidiques, notamment en raison du tératogènes décrits de la metformine et des sulfa- risque de cétogenèse secondaire qui pourrait être mides hypoglycémiants. Fibrates et statines sont délétère pour le développement neurologique du contre-indiqués. fœtus. • E  : L'insulinothérapie est commencée en cas Question 12 d'échec des mesures hygiéno-diététiques. Réponse : A, B, C, D, E (PMZ). Question 8 Commentaire : Voici les toutes dernières recomman- Réponse : A (PMZ). 498 Commentaire : dations (SFD-SFO, 2016) : • B  : L'objectif est ≤ 1,20  g/l 2  heures après les • La grossesse est un facteur de risque connu de repas. développement et de progression de la rétinopa- • C : Seule l'insuline a l'AMM pendant la grossesse ; thie diabétique (RD). • Dans le diabète gestationnel avéré avec une glycé- elle ne traverse pas la barrière hématoplacentaire. mie normale avant la conception ou dans le pre- • D : Une insuline mixte n'est pas indiquée avant le mier trimestre, le suivi ophtalmologique n'est pas nécessaire. dîner car elle générerait des hypoglycémies post- • Le dépistage de la RD avant la conception est un prandiales. On lui préférera une insuline semi-lente élément nécessaire pour autoriser une grossesse. (NPH) ou par detemir (analogue lent). La glargine • Le suivi doit être au minimum trimestriel pendant n'a pas l'AMM mais a l'autorisation dans les men- la grossesse. tions spéciales. • La surveillance ophtalmologique doit être men- • E : L'HbA1c reflète l'équilibre glycémique des 2-3 suelle chez les patientes avec un facteur de risque derniers mois. De ce fait, le déséquilibre récent de progression de RD  : RD préexistante, HTA, sera « dilué ». En outre, l'hémodilution rend néphropathie, longue durée de diabète, absence difficile l'interprétation de l'HbA1c pendant la de programmation de la grossesse, baisse de grossesse. l'HbA1c au premier trimestre et ce quelle que soit Question 9 l'HbA1c à la conception. Réponse : B (PMZ). • Une RD préproliférante ou une RD proliférante Commentaire : avant la grossesse est une contre-indication tem- • A, E  : Pas de césarienne ni de déclenchement poraire à une grossesse et une indication au laser systématique ; notamment, l'insulinothérapie en avant le début de grossesse. soi n'est pas une indication à la césarienne, mais • Il n'est pas recommandé de traiter un œdème maculaire pendant la grossesse. plutôt le déséquilibre glycémique persistant : pour mémoire, le glucose passe la barrière hématopla- • La RD ne peut être qu'une indication exception- nelle à une interruption de grossesse. centaire mais pas l'insuline maternelle. • D : Idem pour les hypoglycémies néonatales dues à un hyperinsulinisme fœtal secondaire à une hyper- Question 13 Réponse : A, B, C. glycémie maternelle (et donc fœtale). • C : L'allaitement est recommandé : il semble avoir une action bénéfique sur le risque de diabète de Question 14 type 2 maternel après son diabète gestationnel, et Réponse : C, D. protéger en partie le futur enfant d'une prise de Commentaire : • A : Pas moins de 1 600 kcal par jour, aux deuxième poids plus importante. et troisième trimestres notamment. Question 10 • B  : Éviter une restriction glucidique trop impor- Réponse : A, B. tante, qui pourrait être responsable d'une Commentaire  : Le risque de diabète est classique- cétose ; la cétose a été associée avec un moindre ment de 50  % à 10  ans et est encore plus impor- développement neurologique dans une étude tant en cas d'autres facteurs de risque, dont ici ancienne.

Cas cliniques 25 • C, D : Besoins en insuline modifiés : diminution en Question 3 499 début de grossesse, puis augmentation vers 24 SA Réponse : C. (comme dans le diabète gestationnel), puis chute brutale après l'accouchement. Question 4 Réponse : B. • E : L'acide folique est recommandé avant et pen- dant le premier trimestre pour diminuer le risque Question 5 de malformation neurologique (après le premier Réponse : B, C, E. trimestre : organogenèse terminée). Commentaire : La densitométrie osseuse est un exa- men radiologique qui dose l'hydroxyapatite présent Question 15 dans les pièces osseuses de référence. Ce dosage bio- Réponse : A, B, C, D. logique est relié statistiquement au risque de fracture ostéoporotique. Grille d'évaluation PMZ Entraînement C Question 6 No Réponses Réponse : A, B, D. 1C B 2 A, B A Question 7 3B C Réponse : A, C, D. 4A Commentaire : 5C C • hypercalcémie + hyperparathyroïdie = hyperpara- 6 A, D A 7C B thyroïdie primaire ; 8A • l'ostéoporose cortico-induite est indiscutable ; 9B E • maladie de Cushing. 10 A, B 11 B, D, E Question 8 12 A, B, C, D, E Réponse : B. 13 A, B, C 14 C, D Question 9 15 A, B, C, D Réponse : C, D. Commentaire  : La désoxypiridinoline, les CTX et les Total NTX sont des fragments de dégradation du collagène de type 1 qui est le principal constituant protéique de Cas clinique 5 la matrice osseuse. Question 1 Question 10 Réponse : A, C. Réponse : A, B, C, D, E. Commentaire : La maladie de Cushing est un hyper- cortisolisme métabolique ACTH-dépendant d'origine Question 11 hypophysaire. Réponse : A, D, E. Commentaire : Le traitement de l'ostéoporose cor- Question 2 tisonique prend en compte la part d'ostéomalacie Réponse : A, B, D. qui lui est associée. La compensation de la carence Commentaire  : L'hypercortisolisme métabolique est en vitamine  D dépend du taux plasmatique pré- responsable d'une usure physiologique accélérée car thérapeutique ; 100 000  UI par mois est un bon il combine les effets glucocorticoïdes (accélération compromis. Une posologie de 100 000 unités tous du catabolisme protidique, athérogenèse) et miné- les 3 mois est inadaptée car elle correspond à une ralocorticoïde (rétention hydrosodée et hypertension substitution simple en dehors de tout contexte artérielle). pathologique. Question 12 Réponse : D. Commentaire : • le raloxifène à une action antiostéoporotique insuf- fisante et est contre-indiqué chez l'homme ; • l'acide zolédronique en perfusion annuelle est une alternative utilisée en relais du tériparatide ; • le dénosumab en injection semestrielle est égale- ment une alternative, mais il doit être réservé après un premier traitement par bisphosphonates ou en cas d'insuffisance rénale ;

Cas cliniques • le tél'roipsatéraotpidoero(sPeTHc1o-3r4t)iseosnt iqleuetr,aidteamnsenlta de choix Grille d'évaluation de mesure No Réponses où le patient présente des fractures vertébrales 1 B, C 2 A, D prévalentes ; 3 A, B • le risédronate oral bimensuel reste une alternative de deuxième ligne. Grille d'évaluation 4 A, B, C, D, E 5 B, E No Réponses Points PMZ SMZ 6 A, D 1 A, C 8 (4, 4) A 7 A, B, E 2 A, B, D 9 (3, 3, 3) 8 A 3C 7 B 9 A, D 4B 7 10 B, D 5 B, C, E 9 (3, 3, 3) A Total 6 A, B, D 9 (3, 3, 3) A 7 A, C, D 10 (3, 4, 3) 8B 7 Cas clinique 7 9 C, D 8 (4, 4) Question 1 Réponse : C, D. 10 A, B, C, D, E 10 (2, 2, 2, 2, 2) Commentaire  : Phéochromocytome, adénome, car- cinome sécrétant du cortisol et adénome de Cohn 11 A, D, E 9 (3, 3, 3) peuvent entraîner une hypertension artérielle d'ori- gine surrénalienne. Néanmoins, les caractéristiques 12 D 7 de la masse qui est volumineuse, qui n'a pas une den- sité spontanée compatible avec un contenu graisseux Total 100 (< 10 UH) et a un caractère très hétérogène, ne sont 500 pas compatibles avec un adénome de Cohn ou un adénome cortisolique. Cas clinique 6 Question 2 Question 1 Réponse : A, B, C, D, E. Réponse : B, C. Commentaire  : Les symptômes du phéochromocy- tome sont essentiellement des effets vasomoteurs liés Question 2 à la sécrétion de catécholamines, dont des complica- Réponse : A, D. tions cardiovasculaires. Néanmoins, les phéochromo- cytomes possèdent une machinerie cellulaire capable Question 3 de synthétiser peptides et interleukines et peuvent Réponse : A, B. être responsables de multiples syndromes paranéo- plasiques (syndrome inflammatoire, hypercalcémie Question 4 par sécrétion de PTH-related peptide, etc.). Il est par Réponse : A, B, C, D, E. ailleurs de plus en plus décrit des phéochromocytomes découverts fortuitement, qui sont totalement asymp- Question 5 tomatiques, liés à une sécrétion minime de catécho- Réponse : B, E. lamines. La recherche d'un phéochromocytome doit donc être envisagée devant tout incidentalome qui Question 6 ne possède pas des caractéristiques d'adénome au Réponse : A, D. scanner. Question 7 Question 3 Réponse : A, B, E. Réponse : D, E. Commentaire  : Les catécholamines plasmatiques Question 8 ont une demi-vie brève et disparaissent rapidement Réponse : A. du plasma après un épisode sécrétoire, alors que Question 9 Réponse : A, D. Question 10 Réponse : B, D.

Cas cliniques 25 les métanéphrines urinaires ou plasmatiques ont au décours de l'intervention et, en l'absence de syn- 501 une demi-vie longue et sont synthétisées en perma- drome de prédisposition génétique, dans environ nence par le phéochromocytome. Le diagnostic de 10 % des cas. Il est donc indispensable de réaliser un phéochromocytome repose donc sur la mesure des suivi prolongé des patients pendant au moins 10 ans, métanéphrines. Les performances des métanéphrines du fait de l'évolution parfois lente des tumeurs neu- plasmatiques ou urinaires sont globalement sem- roendocrines. Celui-ci se fera avant tout par un exa- blables. La chromogranine A est un marqueur non men clinique et un dosage annuel des métanéphrines. spécifique des vésicules d'exocytose et peut être éle- Environ un tiers des phéochromocytomes d'appa- vée en cas de phéochromocytome ; néanmoins, elle rence sporadique répondent en fait à un détermi- constitue un marqueur de deuxième intention utili- nisme génétique (mutation germinale). Il est donc sable en cas de valeur de métanéphrines plasmatiques indispensable de réaliser une enquête génétique ou urinaires « limite ». chez tout patient, et ce d'autant plus que celui-ci est jeune. De multiples gènes sont rendus respon- Question 4 sables de cette anomalie, notamment le gène RET Réponse  : Oui, car l'hypertension survient chez un (néoplasie endocrinienne multiple de type  2) mais, homme jeune. plus fréquemment, les mutations des gènes codant les sous-unités B et D de la succinate déshydrogé- Question 5 nase (SDHB et SDHD), responsables du syndrome Réponse : B, D, E. phéochromocytome-paragangliomes. Commentaire  : Un bilan complémentaire du reten- tissement du phéochromocytome est indispensable, Grille d'évaluation Points Entraînement notamment de son retentissement cardiovasculaire 5 avec, au minimum, un électrocardiogramme et la No Réponses 10 réalisation d'une échographie cardiaque du fait des 1 C, D 7 effets toxiques des catécholamines sur le myocarde. 2 A, B, C, D, E 5 Des examens d'imagerie morphofonctionnelle 3 D, E 10 peuvent être utiles afin de préciser les caractéristiques 4 Oui 5 du phéochromocytome, d'éventuellement évaluer 5 B, D, E 8 son extension ou l'existence de métastases. Dans ce 6 B, D, E 50 cas de figure, la scintigraphie à la MIBG et le PET-scan 7 B, C, D au FDG sont des examens utiles, mais leur réalisation n'est cependant pas systématique. Il convient de ne Total pas confondre la MIBG avec le MIBI qui est un traceur qui permet de visualiser les adénomes parathyroïdiens. Cas clinique 8 Question 6 Question 1 Réponse : B, D, E. Réponse : B, D, E. Commentaire  : La chirurgie du phéochromocytome est une chirurgie difficile, surtout par rapport aux Question 2 modifications hémodynamiques peropératoires enga- Réponse : A, C, E. gées par un relargage massif de catécholamines. Une préparation préopératoire est recommandée et celle- Question 3 ci repose sur les alphabloquants qui antagonisent les Réponse : A, C. effets vasoconstricteurs des catécholamines et non sur les bêtabloquants qui peuvent au contraire démasquer Question 4 l'effet alpha et majorer l'hypertension artérielle. Le type Réponse : A, B, D. de voie d'abord est étudié au cas par cas, en sachant que le plus souvent, pour les phéochromocytomes de Question 5 taille modérée, la chirurgie laparoscopique est utilisée. Réponse : A, B, E. Après l'ablation de la tumeur, la chute brutale du taux de catécholamines peut effectivement entraîner Question 6 l'hypotension et l'hypoglycémie. Réponse : B, C, D. Commentaire  : Une cortisolémie basale à 8  h du Question 7 matin dans les valeurs basses de la normale élimine Réponse : B, C, D. une insuffisance surrénalienne patente, mais n'élimine Commentaire  : L'étude anatomopathologie ne pas une insuffisance surrénalienne latente, c'est-à-dire permet pas de faire le diagnostic de malignité du l'impossibilité pour l'axe corticotrope d'augmenter très phéochromocytome, qui ne peut être affirmée que significativement la production de cortisol en réponse devant la présence d'un envahissement régional ou de métastases. Des métastases peuvent apparaître

Cas cliniques à une affection intercurrente (stress aigu psycholo- n'est pas la situation du cas clinique. Points gique ou somatique). Dans le contexte d'un patient Question 14 ayant pris des corticoïdes et donc d'une insuffisance Réponse : B, C, D. corticotrope secondaire, on peut utiliser des tests Question 15 de référence qui « interrogent » directement le sys- Réponse : B, E. tème hypothalamo-hypophysaire. C'est le cas du test d'hypoglycémie insulinique. Le test au Synacthène®, Grille d'évaluation qui consiste à administrer un analogue synthétique No Réponses de l'ACTH et à stimuler le cortex surrénalien, donne une évaluation indirecte des structures supérieures de l'axe corticotrope. En effet, en cas d'insuffisance 1 B, D, E 4 corticotrope prolongée du fait de l'absence de stimu- 2 A, C, E 8 lation chronique par l'ACTH, il s'ensuit une atrophie 3 A, C 5 corticosurrénalienne au bout de quelques mois. Lors 4 A, B, D 6 de la stimulation aiguë par le Synacthène®, il y aura 5 A, B, E 9 une réponse de la corticosurrénale saine mais qui, du 6 B, C, D 9 fait de son atrophie, sera insuffisante. La cortisolémie 7C 5 ne dépassera pas le seuil de 500 nmol/l, témoignant 8D 6 donc de l'incapacité de l'axe corticotrope à répondre à 9C 9 un stimulus aigu exogène. En cas d'insuffisance surré- 10 C, E 8 nalienne périphérique, le cortex surrénalien étant déjà stimulé au maximum par l'ACTH endogène, l'adminis- tration du Synacthène® n'entraîne en général aucune augmentation de la cortisolémie. Question 7 11 Éducation thérapeutique sur les 4 Réponse : C. circonstances de survenue de la Question 8 décompensation aiguë, ses symptômes 502 Réponse : D. annonciateurs et l'attitude à adopter Question 9 12 A, D 8 Réponse : C. 13 A, C 5 Commentaire : Chez le sujet jeune, les adénomes hypo- 14 B, C, D 8 physaires ne sont pas les tumeurs les plus fréquentes de la région sellaire et, dans le contexte d'une atteinte 15 B, E 6 également post-hypophysaire (diabète insipide), il faut Total 100 songer au craniopharyngiome qui est la tumeur encé- phalique la plus fréquente de l'enfant et a un aspect Cas clinique 9 IRM caractéristique d'alternance de zones charnues et de zones liquidiennes visibles en T2. Des calcifications Question 1 seraient vraisemblablement visibles au scanner. Réponse : A, B, C, D. Question 10 Question 2 Réponse : C, E. Réponse : A, D. Question 11 Question 3 Réponse  : éducation thérapeutique sur les circons- Réponse : C. tances de survenue de la décompensation aiguë, ses symptômes annonciateurs et l'attitude à adopter. Question 4 Question 12 Réponse : D. Réponse : A, D. Question 5 Question 13 Réponse : A, D, E. Réponse : A, C. Commentaire  : La patiente est susceptible de déve- Question 6 Réponse : A, C. lopper d'autres atteintes endocrines auto-immunes Question 7 s'intégrant dans le cadre d'une polyendocrinopathie Réponse : A, C. auto-immune de type  2. L'hypoparathyroïdie auto- immune peut faire partie d'une polyendocrinopathie Question 8 auto-immune mais dite de type 1 (mutation du gène Réponse : A, B. AIRE), qui survient tôt dans l'enfance et s'accom- pagne entre autres d'une candidose cutanée, ce qui Question 9 Réponse : A, E.

Cas cliniques 25 Question 10 Question 15 503 Réponse : A. Réponse : C, D. Grille d'évaluation Question 16 Réponse : B, C, D. No Réponses Points PMZ SMZ Grille d'évaluation 1 A, B, C, D 5 (2, 1,1 1) A No Réponses Points PMZ SMZ 2 A, D 10 (4, 3, 3) C 1 C, D 6 C 2B 6 A, C 3C 10 E 3 A, C, D 7 A, B, D C 4 B, C, D, E 7 D, E 4D 10 5 B, D, E 6 A 5 A, D, E 15 (5, 5, 5) C, D 6 A, C, D 6 7 C, E 6 C 6 A, C 10 (5, 5) 8 A, B, C 6 7 A, C 10 (5, 5) 9 B, D 6 10 C, D, E 6 8 A, B 10 (5, 5) 11 B, D 7 12 D, E 6 9 A, E 10 (5, 5) 13 C 7 10 A 10 14 A, C 6 15 C, D 6 Total 100 16 B, C, D 6 100 Cas clinique 10 Total Entraînement Question 1 Cas clinique 11 Réponse : C, D. Question 1 Question 2 Réponse : A, C. Réponse : B. Question 2 Question 3 Réponse : A, D. Réponse : A, C, D. Question 3 Question 4 Réponse : C. Réponse : B, C, D, E. Question 4 Question 5 Réponse : E. Réponse : B, D, E. Question 5 Question 6 Réponse : A, B, C, D, E. Réponse : A, C, D. Question 6 Question 7 Réponse : A, C, E. Réponse : C, E. Question 7 Question 8 Réponse : C, D, E. Réponse : A, B, C. Question 8 Question 9 Réponse : C. Réponse : B, D. Question 9 Question 10 Réponse : A, B, D, E. Réponse : C, D, E. Question 10 Question 11 Réponse : E. Réponse : B, C. Question 12 Réponse : D, E. Question 13 Réponse : C. Question 14 Réponse : A, C.

Cas cliniques Question 11 Question 5 Réponse : B, D, E. Réponse : A, C. Question 12 Question 6 Réponse : C, D. Réponse : B, C, D. Question 13 Question 7 Réponse : C, D, E. Réponse : C. Question 14 Réponses Question 8 Réponse : A. Réponse : E. Question 15 Commentaire : Réponse : C, D. • B : Objectif LDL-cholestérol < 1 g/l ou même 0,7 g/l Grille d'évaluation (en prévention secondaire, donc il faut augmenter No la dose de la statine). • C : La dose de metformine est réduite à demi-dose maximum quand le DFGe est entre 30 et 60  ml/ min. 1 A, C Question 9 2 A, D Réponse : D. 3 C Commentaire : 4 E • E : Recommandation : 150 min./semaine. 5 A, B, C, D, E • C  : Recommandation  : deux à trois pour les 6 A, C, E 7 C, D, E hommes. 8 C • A  : Cible 7  %. Rappel HAS  : « Pour les patients 504 9 A, B, D, E 10 E avec un antécédent de complication macrovascu- 11 B, D, E laire considérée comme non évoluée, un objectif 12 C, D inférieur ou égal à 7 % est recommandé. » 13 C, D, E 14 A Question 10 Réponse : A, B, C, D, E. Question 11 Réponse : A, D, E. Commentaire : • B : Objectif < 0,8 g/kg par jour. • C  : Plus de recommandation sur les apports ali- mentaires en cholestérol. 15 C, D Question 12 Total Réponse : B, C, D. Commentaire  : Le risque cardiovasculaire est bien supérieur au risque rénal. Cas clinique 12 Question 13 Réponse : A, B, C. Question 1 Commentaire : Réponse : A, D, E. • D : La metformine n'est pas néphrotoxique. Commentaire : • E  : Le ramipril est indiqué en cas d'altération de • C  : N'est pas recommandé dans le parcours de fonction rénale. soins pour un sujet ni avec diabète de type  2, ni avec coronaropathie. Question 14 Réponse : A, B, D, E. Question 2 Commentaire  : Le patient a un stent ; il a un traite- Réponse : C, E. ment anti-agrégant simple qui doit être poursuivi. Question 3 Question 15 Réponse : A, B, C, D, E. Réponse : A, B, C. Commentaire : Commentaire : • A, C : Directement en rapport. • D  : Il y a moins d'un point d'écart entre l'HbA1c • B, D, E  : Justifiés également dans le contexte cli- présente et la cible, donc plutôt pas de GLP-1R. nique présenté ici. • A, E : Pas d'indication à mettre à la fois un sulfo- Question 4 nylurée (gliclazide) et un glinide (même mécanisme Réponse : B, D, E. d'action).

Cas cliniques 25 Grille d'évaluation est de plus de 1 % : on ne doit donc pas utiliser un 505 inhibiteur de DPP-4. L'IMC ne dépasse pas 30 : on ne No Réponses Points PMZ SMZ doit donc pas utiliser un analogue de GLP-1. A, D 1 A, D, E 10 Question 6 Réponse : B, D. 2 C, E 5 Commentaire  : La neuropathie et ses symptômes ne régressent pas avec l'équilibre du diabète ; celui- 3 A, B, C, D, E 5 ci ralentit la progression des complications de la 4 B, D, E 5 microangiopathie. 5 A, C 5 Question 7 Réponse : B, C. 6 B, C, D 10 Commentaire  : L'ALD 30 ne donne pas de droit concernant les consultations de podologie. Le patient 7C 10 a une neuropathie avec des déformations ; il a donc un grade 2 de risque, ce qui lui donne droit à quatre 8E 10 consultations remboursées par an. 9D 8 Question 8 10 A, B, C, D, E 7 Réponse : A, B, C, E. Commentaire  : Le coricide est contre-indiqué car 11 A, D, E 5 source de plaie. 12 B, C, D 5 Question 9 13 A, B, C 5 Réponse : C, D, E. 14 A, B, D, E 5 Commentaire : Le diagnostic d'infection est clinique. Sans signes d'inflammation, pas d'infection, donc il 15 A, B, C 5 n'est pas nécessaire de l'hospitaliser ni de prescrire des antibiotiques. Le traitement comporte un débride­ Total 100 ment, la suppression de l'appui avec une chaussure de décharge et le repos. Cas clinique 13 Entraînement Question 10 Question 1 Réponse : A, B, C. Réponse : A, C, D, E. Commentaire : La thyroïdite de Hashimoto donne une Commentaire  : La neuropathie douloureuse hypothyroïdie. Dans le syndrome d'apnées du som- touche les petites fibres et peut s'accompagner meil, il n'y a pas de manifestations adrénergiques en de tous ces signes. Les réflexes achilléens peuvent période de veille. être abolis. Question 11 Question 2 Réponse : C, E. Réponse : A, B, C, D, E. Commentaires : • B : La bonne réponse est « inférieure à 10 ». Question 3 • D : il y a au contraire disparition de la bradycardie Réponse : B, C. réflexe. Question 4 Réponse : A, D. Question 12 Commentaire  : La neuropathie ne présente pas Réponse : A, B, C, D. d'atypies. Il n'y a pas d'indication à faire un EMG ni Commentaire : Le syndrome du canal carpien résulte une biopsie. Le fond d'œil annuel est justifié pour d'une compression du nerf médian et pas d'une la surveillance de la rétinopathie. L'HbA1c doit être atteinte des fibres végétatives. dosée 3 à 4 fois par an. L'artériographie est réser- vée aux indications cliniques de revascularisation Question 13 (ischémie critique, artériopathie sévère avec trouble Réponse : C, D. trophique). Commentaire : L'effet secondaire digestif le plus fré- quent de la metformine est la diarrhée. Question 5 Réponse : E. Question 14 Commentaire : Recommandations HAS sur la prise en Réponse : A, B, C. charge thérapeutique du diabète de type 2. Le patient a une insuffisance rénale modérée ; on ne peut donc Question 15 pas augmenter la metformine ni le glimépiride (sul- Réponse : B, C, D. famide hypoglycémiant). Le gain d'HbA1c à obtenir

Cas cliniques Grille d'évaluation Question 13 Réponse : B. No Réponses Points PMZ SMZ A,D B Question 14 1 A, C, D, E 8 Réponse : A, B, C, D. A 2 A, B, C, D, E 6 C,E Question 15 Réponse : B, D, E. 3 B, C 8 Question 16 4 A, D 8 Réponse : A, D, E. 5E 6 6 B, D 6 7 B, C 8C Grille d'évaluation No Réponses Points 8 A, B, C, E 6 D PMZ SMZ 9 C, D, E 8 D 1B 8B 10 A, B, C 6 2 D, E 7 D, E 11 C, E 6 3 B, C, E 8 B, E A, D 12 A, B, C, D 6 4 A, B, C, D, E 7 13 C, D 6D 5 B, C, E 7 A, D 14 A, B, C 6 6B 8B 15 B, C, D 6 7 C, E 6 C Total 100 8 B, D 5 9 A, C, D 6 B, E Cas clinique 14 10 A, B, C, D, E 7 506 11 A, B, C, D 6 Question 1 Réponse : B. 12 A, B, C, D, E 5 B, D A Question 2 13 B 4 B A, E Réponse : D, E. 14 A, B, C, D 5 B, C Question 3 Réponse : B, C, E. 15 B, D, E 5 B, E A Question 4 16 A, D, E 6 B Réponse : A, B, C, D, E. Total 100 Question 5 Cas clinique 15 Réponse : B, C, E. Question 6 Question 1 Réponse : B. Réponse : A, B, C. Question 7 Question 2 Réponse : C, E. Réponse : A, B, C, D, E. Question 8 Question 3 Réponse : B, D. Réponse : B, D. Question 9 Question 4 Réponse : A, C, D. Réponse : A. Question 10 Question 5 Réponse : A, B, C, D, E. Réponse : C, D. Question 11 Question 6 Réponse : A, B, C, D. Réponse : A, E. Question 12 Question 7 Réponse : A, B, C, D, E. Réponse : D.

Cas cliniques 25 Question 8 Commentaire : 507 Réponse : A, B, C, D, E. • A  : En cas d'hyperglycémie modérée à jeun  : Question 9 dosage un an plus tard. Réponse : E. • B : HGPO possible. • C : On affirme le diagnostic de diabète seulement Question 10 Réponse : C, D. en cas de signes cardinaux associés à une hypergly- cémie > 2 g/l. Question 11 • E : Utilisée pour le dépistage. Réponse : D, E. Question 3 Question 12 Réponse : B, E. Réponse : D. Commentaire : • A : 6,5 %. Question 13 • C : 135 mm Hg en automesure. Réponse : A, C, D. • D  : Objectif =  normoalbuminurie (< 30  mg par Question 14 24 heures). Réponse : A, B, D. Question 4 Question 15 Réponse : A, C, E. Réponse : B, C. Commentaire : Traitement initial en monothérapie par metformine et poursuite des modifications thérapeu- Grille d'évaluation tiques du mode de vie. No Réponses Points PMZ SMZ Question 5 Entraînement 1 A, B, C 5 D,E Réponse : B. 2 A, B, C, D, E 10 A, B, C Commentaire : 3 B, D 5 B, C, D • E : Effet sur l'AMPK. 4A 10 D A 5 C, D 5 E Question 6 6 A, E 5 C, D D Réponse : B, E. 7D 5 E B Commentaire : 8 A, B, C, D, E 5 D C • D : IMC < 35. 9E 10 A • E  : Possible, même s'il existe des thérapeutiques 10 C, D 5 A, B E 11 D, E 5 orales possibles (et souvent privilégiées par les 12 D 10 patients). 13 A, C, D 10 14 A, B, D 10 Question 7 15 B, C 5 Réponse : A, C, D. 100 Commentaire : Total • B : L'effet fréquent est l'hypoglycémie. • E : C'est sur le canal potassique. Cas clinique 16 Question 8 Question 1 Réponse : A, B, C. Réponse : A, B, D, E. Commentaire : Commentaire  : Recommandation HAS « Parcours de • D : Le stress augmente la glycémie. soins du sujet diabète de type 2 adulte » (mars 2014). • E : Une activité moindre induit une glycémie plus • C : Âge < 45 ans. forte. Question 2 Réponse : B, D. Question 9 Réponse : A, B, D, E. Commentaire : • C  : La ménopause ne s'associe pas à ce type de difficulté métabolique. Question 10 Réponse : A, C, E. Commentaire : • B : C'est la metformine. • D : Médicaments anciens peu chers. Question 11 Réponse : A, C, D, E.


Like this book? You can publish your book online for free in a few minutes!
Create your own flipbook